You are on page 1of 414

INTERMEDIATE 2 – ADDITIONAL QUESTION BANK

UNIT 1 :
Straight Line Percentages

Volumes Brackets

The Circle Factors

EXIT
INTERMEDIATE 2 – ADDITIONAL QUESTION BANK

You have chosen to study:

UNIT 1 :
Straight Line

Please choose a question to attempt from the following:

1 2 3 4

Back to
EXIT Unit 1 Menu
Question

Begin Solution
Continue Solution
Comments

Menu

Back to Home
Comments

Next Comment

Menu

Back to Home
STRAIGHT LINE : Question 1
Find the equation of the line shown in this diagram.

Get hint

Reveal answer only

Go to full solution

Go to Comments

Go to Straight Line Menu

EXIT
STRAIGHT LINE : Question 1
Find the equation of the line shown in this diagram.
2. Find
4.
3. Substitute
1. IdentifyNote
two
(0, 9) gradient
into using
y = mx + c
pointswhere
on the
gradient
line
line.
formula.
cuts y-
What would you like to do now?
axis
Reveal answer only

Go to full solution

(3, 0) Go to Comments

Go to Straight Line Menu

EXIT
STRAIGHT LINE : Question 1
Find the equation of the line shown in this diagram.

What would you like to do now?

Try another like this

Go to full solution

Go to Comments

Go to Straight Line Menu

y = −3 x + 9
EXIT
Question 1 1. Identify two points on the line.
Find the equation of this line
2. Find gradient using gradient formula.
(0, 9)
Using (0, 9) and (3, 0):

y2 − y1 0−9 −9
m= = = =-3
(3, 0)
x2 − x1 3−0 3
3. Note where line cuts y-axis.

Cuts y-axis at (0, 9) so c = 9

4. Substitute into y = mx + c .
Continue Solution
Try another like this y = mx + c
Comments becomes y = −3 x + 9
Straight Line Menu
What would you like to do now?
Back to Home
Comments
• Must learn gradient formula:
1. Identify two points on the line.

2. Find gradient using gradient formula.


B(x2,y2)
Using (0, 9) and (3, 0):

y2 − y1 0−9 −9
m= = = =-3 A(x1,y1)
x2 − x1 3−0 3
y2 – y 1
3. Note where line cuts y-axis. mAB =
x2 – x1
Cuts y-axis at (0, 9) so c = 9

4. Substitute into y = mx + c .

y = mx + c Next Comment

becomes y = −3 x + 9 Straight Line Menu

Back to Home
Comments

• Must learn equation of the line:


1. Identify two points on the line.

2. Find gradient using gradient formula. Y = mx + c


Using (0, 9) and (3, 0):

y2 − y1 0−9 −9 gradient y - intercept


m= = = =-3
x2 − x1 3−0 3
3. Note where line cuts y-axis.

Cuts y-axis at (0, 9) so c = 9

4. Substitute into y = mx + c .

y = mx + c Next Comment

becomes y = −3 x + 9 Straight Line Menu

Back to Home
Comments

• Line slopes downward from


1. Identify two points on the line. left to right

Negative gradient
2. Find gradient using gradient formula.
m -ve
Using (0, 9) and (3, 0):

y2 − y1 0−9 −9
m= = = =-3 • Line slopes upwards from
x2 − x1 3−0 3 left to right

3. Note where line cuts y-axis. Positive gradient

Cuts y-axis at (0, 9) so c = 9 m +ve

4. Substitute into y = mx + c .

y = mx + c Next Comment

becomes y = −3 x + 9 Straight Line Menu

Back to Home
STRAIGHT LINE : Question 1B
Find the equation of the line shown in this diagram.

Get hint

Reveal answer only

Go to full solution

Go to Comments

Go to Straight Line Menu

EXIT
STRAIGHT LINE : Question 1B
Find the equation of the line shown in this diagram.
2.4.Find
1. Identify Substitute
two3. Note
gradient
points on yusing
intothe = mx + c
where
gradientline
line.
formula. cuts y-
What would you like to do now?
axis
Reveal answer only

(2, 0)
Go to full solution

Go to Comments
(0,-1)
Go to Straight Line Menu

EXIT
STRAIGHT LINE : Question 1B
Find the equation of the line shown in this diagram.

What would you like to do now?

Go to full solution

Go to Comments

Go to Straight Line Menu

EXIT y = 12 x − 1
Question 1B 1. Identify two points on the line.
Find the equation of this line
2. Find gradient using gradient formula.

Using (0, -1) and (2, 0):

(2, 0) y2 − y1 0 − (−1) 1
m= = =
x2 − x1 2−0 2
(0,-1)
3. Note where line cuts y-axis.

Cuts y-axis at (0, -1) so c = -1

4. Substitute into y = mx + c .
Begin Solution
Continue Solution y = mx + c
Comments becomes
y = 12 x − 1
Straight Line Menu
What would you like to do now?
Back to Home
Comments
• Must learn gradient formula:
1. Identify two points on the line.

2. Find gradient using gradient formula.


B(x2,y2)
Using (0, -1) and (2, 0):

y2 − y1 0 − (−1) 1
m= = =
x2 − x1 2−0 2 A(x1,y1)

y2 – y 1
3. Note where line cuts y-axis. mAB =
x2 – x1
Cuts y-axis at (0, -1) so c = -1

4. Substitute into y = mx + c .

y = mx + c Next Comment

becomes
y = 12 x − 1 Straight Line Menu

Back to Home
Comments

• Must learn equation of the line:


1. Identify two points on the line.

2. Find gradient using gradient formula. Y = mx + c


Using (0, -1) and (2, 0):

y2 − y1 0 − (−1) 1 gradient y - intercept


m= = =
x2 − x1 2−0 2
3. Note where line cuts y-axis.

Cuts y-axis at (0, -1) so c = -1

4. Substitute into y = mx + c .

y = mx + c Next Comment

becomes
y = 12 x − 1 Straight Line Menu

Back to Home
Comments

• Line slopes downward from


1. Identify two points on the line. left to right

2. Find gradient using gradient formula. Negative gradient

Using (0, -1) and (2, 0): m -ve

y2 − y1 0 − (−1) 1
m= = =
x2 − x1 2−0 2 • Line slopes upwards from
left to right
3. Note where line cuts y-axis. Positive gradient

Cuts y-axis at (0, -1) so c = -1 m +ve


4. Substitute into y = mx + c .

y = mx + c Next Comment

becomes
y = 12 x − 1 Straight Line Menu

Back to Home
STRAIGHT LINE : Question 2
Find the equation of the line shown in this diagram.

Get hint

Reveal answer only

Go to full solution

Go to Comments

Go to Straight Line Menu

EXIT
STRAIGHT LINE : Question 2
Find the equation of the line shown in this diagram.

(6, 60) What would you like to do now?

Reveal answer only

Go to full solution

Go to Comments
(0, 30) 2.
4. Find
Substitute
3. Note
1. Identify two
gradienty using
points on into
the =where
mx + c Go to Straight Line Menu
gradientline
line.
formula.cuts y-
(NOTE
SCALES) axis

EXIT
STRAIGHT LINE : Question 2
Find the equation of the line shown in this diagram.

What would you like to do now?

Try another like this

Go to full solution

Go to Comments

Go to Straight Line Menu

y = 5 x + 30
EXIT
Question 2 1. Identify two points on the line.
Find the equation of this line
2. Find gradient using gradient formula.
(6, 60)
Using (0, 30) and (6, 60):

y2 − y1 60 − 30 30
m= = = =5
x2 − x1 6−0 6
(0, 30)
3. Note where line cuts y-axis.

Cuts y-axis at (0, 30) so c = 30

4. Substitute into y = mx + c .
Continue Solution
Try another like this y = mx + c
Comments becomes
y = 5 x + 30
Straight Line Menu
What would you like to do now?
Back to Home
Comments
• Must learn gradient formula:
1. Identify two points on the line.

2. Find gradient using gradient formula.


B(x2,y2)
Using (0, 30) and (6, 60):

y2 − y1 60 − 30 30
m= = = =5
x2 − x1 6−0 6 A(x1,y1)

y2 – y 1
3. Note where line cuts y-axis. mAB =
x2 – x1
Cuts y-axis at (0, 30) so c = 30

4. Substitute into y = mx + c .

y = mx + c Next Comment

becomes
y = 5 x + 30 Straight Line Menu

Back to Home
Comments

• Must learn equation of the line:


1. Identify two points on the line.

2. Find gradient using gradient formula. Y = mx + c


Using (0, 30) and (6, 60):

y2 − y1 60 − 30 30 gradient y - intercept
m= = = =5
x2 − x1 6−0 6
3. Note where line cuts y-axis.

Cuts y-axis at (0, 30) so c = 30

4. Substitute into y = mx + c .

y = mx + c Next Comment

becomes
y = 5 x + 30 Straight Line Menu

Back to Home
Comments

• Line slopes downward from


1. Identify two points on the line. left to right

2. Find gradient using gradient formula. Negative gradient

Using (0, 30) and (6, 60): m -ve

y2 − y1 60 − 30 30
m= = = =5
x2 − x1 6−0 6 • Line slopes upwards from
left to right
3. Note where line cuts y-axis. Positive gradient

Cuts y-axis at (0, 30) so c = 30 m +ve


4. Substitute into y = mx + c .

y = mx + c Next Comment

becomes
y = 5 x + 30 Straight Line Menu

Back to Home
STRAIGHT LINE : Question 2B
Find the equation of the line shown in this diagram.

Get hint

Reveal answer only

Go to full solution

Go to Comments

Go to Straight Line Menu

EXIT
STRAIGHT LINE : Question 2B
Find the equation of the line shown in this diagram.

3. Note
1. Identify two
2. Find
points where
4.onSubstitute
the What would you like to do now?
gradient using
into
line. line
y = mx + c
gradient
(NOTEcuts y-
formula. Reveal answer only
SCALES) axis

Go to full solution

Go to Comments

Go to Straight Line Menu

EXIT
STRAIGHT LINE : Question 2B
Find the equation of the line shown in this diagram.

What would you like to do now?

Go to full solution

Go to Comments

Go to Straight Line Menu

y = − 18 x + 1
EXIT
Question 2B 1. Identify two points on the line.
Find the equation of this line
(0, 1)
2. Find gradient using gradient formula.

Using (0, 1) and (40, -4):

y2 − y1 −4 − 1 −5 1
m= = = =−
x2 − x1 40 − 0 40 8
(40, -4)
3. Note where line cuts y-axis.

Cuts y-axis at (0, 1) so c = 1

4. Substitute into y = mx + c .
Begin Solution
Continue Solution y = mx + c
Comments becomes
y = − 18 x + 1
Straight Line Menu
What would you like to do now?
Back to Home
Comments
• Must learn gradient formula:
1. Identify two points on the line.

2. Find gradient using gradient formula.


B(x2,y2)
Using (0, 1) and (40, -4):

y2 − y1 −4 − 1 −5 1
m= = = =− A(x1,y1)
x2 − x1 40 − 0 40 8
y2 – y 1
3. Note where line cuts y-axis. mAB =
x2 – x1
Cuts y-axis at (0, 1) so c = 1

4. Substitute into y = mx + c .

y = mx + c Next Comment

y = − x +1
becomes 1 Straight Line Menu
8
Back to Home
Comments

• Must learn equation of the line:


1. Identify two points on the line.

2. Find gradient using gradient formula. Y = mx + c


Using (0, 1) and (40, -4):

y2 − y1 −4 − 1 −5 1 gradient y - intercept
m= = = =−
x2 − x1 40 − 0 40 8
3. Note where line cuts y-axis.

Cuts y-axis at (0, 1) so c = 1

4. Substitute into y = mx + c .

y = mx + c Next Comment

y = − x +1
becomes 1 Straight Line Menu
8
Back to Home
Comments

• Line slopes downward from


1. Identify two points on the line. left to right

Negative gradient
2. Find gradient using gradient formula.
m -ve
Using (0, 1) and (40, -4):

y2 − y1 −4 − 1 −5 1
m= = = =− • Line slopes upwards from
x2 − x1 40 − 0 40 8 left to right

3. Note where line cuts y-axis. Positive gradient

Cuts y-axis at (0, 1) so c = 1 m +ve

4. Substitute into y = mx + c .

y = mx + c Next Comment

y = − x +1
becomes 1 Straight Line Menu
8
Back to Home
STRAIGHT LINE : Question 3
Find the equation of the blue line which is parallel to the red line
shown in this diagram.

Get hint

Reveal answer only

Go to full solution

Go to Comments

Go to Straight Line Menu

EXIT
STRAIGHT LINE : Question 3
Find the equation of the blue line which is parallel to the red line
shown in this diagram.
2. Find
3. two
Note What would you like to do now?
1. Identify 4. Substitute
gradient
where using
points onintothe y = mx + c,
gradient
blue
red line.remembering
formula.
line Reveal answer only
parallel lines
cuts y- (6, 6)
have same
axis Go to full solution
gradient.
(0, 3)
Go to Comments

Go to Straight Line Menu

EXIT
STRAIGHT LINE : Question 3
Find the equation of the blue line which is parallel to the red line
shown in this diagram.

What would you like to do now?

Try another like this

Go to full solution

Go to Comments

Go to Straight Line Menu

y = 12 x − 2
EXIT
Question 3 1. Identify two points on the red line.
Find the equation of blue line
2. Find gradient using gradient formula.
(6, 6)
Using (0, 3) and (6, 6):
(0, 3)
y2 − y1 6−3 3 1
m= = = =
x2 − x1 6−0 6 2
3. Note where blue line cuts y-axis.

Cuts y-axis at (0, -2) so c = -2

4. Substitute into y = mx + c .
Continue Solution
Try another like this y = mx + c
Comments becomes
y = 12 x − 2
Straight Line Menu
What would you like to do now?
Back to Home
Comments
• Must learn gradient formula:
1. Identify two points on the red line.

2. Find gradient using gradient formula.


B(x2,y2)
Using (0, 3) and (6, 6):

y2 − y1 6−3 3 1
m= = = =
x2 − x1 6−0 6 2 A(x1,y1)

y2 – y 1
3. Note where blue line cuts y-axis. mAB =
x2 – x1
Cuts y-axis at (0, -2) so c = -2

4. Substitute into y = mx + c .

y = mx + c Next Comment

becomes
y = 12 x − 2 Straight Line Menu

Back to Home
Comments

• Must learn equation of the line:


1. Identify two points on the red line.

2. Find gradient using gradient formula. Y = mx + c


Using (0, 3) and (6, 6):

y2 − y1 6−3 3 1 gradient y - intercept


m= = = =
x2 − x1 6−0 6 2
3. Note where blue line cuts y-axis.

Cuts y-axis at (0, -2) so c = -2

4. Substitute into y = mx + c .

y = mx + c Next Comment

becomes
y = 12 x − 2 Straight Line Menu

Back to Home
Comments

• Line slopes downward from


1. Identify two points on the red line. left to right

2. Find gradient using gradient formula. Negative gradient

Using (0, 3) and (6, 6): m -ve

y2 − y1 6−3 3 1
m= = = =
x2 − x1 6−0 6 2 • Line slopes upwards from
left to right
3. Note where blue line cuts y-axis. Positive gradient

Cuts y-axis at (0, -2) so c = -2 m +ve


4. Substitute into y = mx + c .

y = mx + c Next Comment

becomes
y = 12 x − 2 Straight Line Menu

Back to Home
Comments

1. Identify two points on the red line. • Parallel lines have equal
gradients
2. Find gradient using gradient formula.
m1 = m2
Using (0, 3) and (6, 6):

y2 − y1 6−3 3 1
m= = = = m1
x2 − x1 6−0 6 2 m2

3. Note where blue line cuts y-axis.

Cuts y-axis at (0, -2) so c = -2

4. Substitute into y = mx + c .

y = mx + c Next Comment

becomes
y = 12 x − 2 Straight Line Menu

Back to Home
STRAIGHT LINE : Question 3B
Find the equation of the blue line which is parallel to the red line
shown in this diagram.

Get hint

Reveal answer only

Go to full solution

Go to Comments

Go to Straight Line Menu

EXIT
STRAIGHT LINE : Question 3B
Find the equation of the blue line which is parallel to the red line
shown in this diagram.
2. Find
3. two
Note
1. Identify 4. Substitute What would you like to do now?
gradient
where using
points onintothe y = mx + c,
gradient
blue
red line.remembering
formula.
line Reveal answer only
parallel lines
cuts y-
have same
axis Go to full solution
gradient.
(0, 80)
Go to Comments

Go to Straight Line Menu

(8, 0)

EXIT
STRAIGHT LINE : Question 3B
Find the equation of the blue line which is parallel to the red line
shown in this diagram.

What would you like to do now?

Go to full solution

Go to Comments

Go to Straight Line Menu

EXIT y = −10 x + 120


Question 3 B 1. Identify two points on the red line.
Find the equation of blue line
2. Find gradient using gradient formula.

Using (0, 80) and (8, 0):

(0, 80) y2 − y1 0 − 80 −80


m= = = = −10
x2 − x1 8−0 8
3. Note where blue line cuts y-axis.
(8, 0)
Cuts y-axis at (0, 120) so c = 120

4. Substitute into y = mx + c .
Begin Solution
Continue Solution y = mx + c
Comments becomes y = −10 x + 120
Straight Line Menu
What would you like to do now?
Back to Home
Comments
• Must learn gradient formula:
1. Identify two points on the red line.

2. Find gradient using gradient formula.


B(x2,y2)
Using (0, 80) and (8, 0):

y2 − y1 0 − 80 −80
m= = = = −10
x2 − x1 8−0 8 A(x1,y1)

y2 – y 1
3. Note where blue line cuts y-axis. mAB =
x2 – x1
Cuts y-axis at (0, 120) so c = 120

4. Substitute into y = mx + c .

y = mx + c Next Comment

becomes y = −10 x + 120 Straight Line Menu

Back to Home
Comments

• Must learn equation of the line:


1. Identify two points on the red line.

2. Find gradient using gradient formula. Y = mx + c


Using (0, 80) and (8, 0):

y2 − y1 0 − 80 −80 gradient y - intercept


m= = = = −10
x2 − x1 8−0 8
3. Note where blue line cuts y-axis.

Cuts y-axis at (0, 120) so c = 120

4. Substitute into y = mx + c .

y = mx + c Next Comment

becomes y = −10 x + 120 Straight Line Menu

Back to Home
Comments

• Line slopes downward from


1. Identify two points on the red line. left to right

2. Find gradient using gradient formula. Negative gradient

Using (0, 80) and (8, 0): m -ve

y2 − y1 0 − 80 −80
m= = = = −10
x2 − x1 8−0 8 • Line slopes upwards from
left to right
3. Note where blue line cuts y-axis. Positive gradient

Cuts y-axis at (0, 120) so c = 120 m +ve


4. Substitute into y = mx + c .

y = mx + c Next Comment

becomes y = −10 x + 120 Straight Line Menu

Back to Home
Comments

1. Identify two points on the red line. • Parallel lines have equal
gradients
2. Find gradient using gradient formula.
m1 = m2
Using (0, 80) and (8, 0):

y2 − y1 0 − 80 −80
m= = = = −10 m1
x2 − x1 8−0 8 m2
3. Note where blue line cuts y-axis.

Cuts y-axis at (0, 120) so c = 120

4. Substitute into y = mx + c .

y = mx + c Next Comment

becomes y = −10 x + 120 Straight Line Menu

Back to Home
STRAIGHT LINE : Question 4
The graph below shows the cost , £C , of hiring a chain-saw for n days.
Find the formula for C in terms of n.

Get hint

Reveal answer only

Go to full solution

Go to Comments

Go to Straight Line Menu

EXIT
STRAIGHT LINE : Question 4
The graph below shows the cost , £C , of hiring a chain-saw for n days.
Find the formula for C in terms of n.
3. Note
1. Identify2.two
Find
pointsgradient
on where
the
4. Substitute What would you like to do now?
using
line. line
intogradient
y = mx + c,
(NOTE
using Ccuts
and n.
formula. Reveal answer only
SCALES)vertical (10, 70)
axis
Go to full solution

Go to Comments

Go to Straight Line Menu


(0, 30)

EXIT
STRAIGHT LINE : Question 4
The graph below shows the cost , £C , of hiring a chain-saw for n days.
Find the formula for C in terms of n.

What would you like to do now?

Try another like this

Go to full solution

Go to Comments

Go to Straight Line Menu

C = 4n + 30
EXIT
Question 4 1. Identify two points on the line.
Find the equation of this line
2. Find gradient using gradient formula.

Using (0, 30) and (10, 70):

y2 − y1 70 − 30 40
(10, 70) m= = = =4
(0, 30) x2 − x1 10 − 0 10
3. Note where line cuts y-axis.

Cuts y-axis at (0, 30) so c = 30

4. Substitute into y = mx + c .
Continue Solution
Try another like this C = mn + c
Comments becomes
C = 4n + 30
Straight Line Menu
What would you like to do now?
Back to Home
Comments
• Must learn gradient formula:
1. Identify two points on the line.

2. Find gradient using gradient formula.


B(x2,y2)
Using (0, 30) and (10, 70):

y2 − y1 70 − 30 40
m= = = =4
x2 − x1 10 − 0 10 A(x1,y1)

y2 – y 1
3. Note where line cuts y-axis. mAB =
x2 – x1
Cuts y-axis at (0, 30) so c = 30

4. Substitute into y = mx + c .

C = mn + c Next Comment

becomes
C = 4n + 30 Straight Line Menu

Back to Home
Comments

• Must learn equation of the line:


1. Identify two points on the line.

2. Find gradient using gradient formula. Y = mx + c


Using (0, 30) and (10, 70):

y2 − y1 70 − 30 40 gradient y - intercept
m= = = =4
x2 − x1 10 − 0 10
3. Note where line cuts y-axis.

Cuts y-axis at (0, 30) so c = 30

4. Substitute into y = mx + c .

C = mn + c Next Comment

becomes
C = 4n + 30 Straight Line Menu

Back to Home
Comments

• Line slopes downward from


1. Identify two points on the line. left to right

2. Find gradient using gradient formula. Negative gradient

Using (0, 30) and (10, 70): m -ve

y2 − y1 70 − 30 40
m= = = =4
x2 − x1 10 − 0 10 • Line slopes upwards from
left to right
3. Note where line cuts y-axis. Positive gradient

Cuts y-axis at (0, 30) so c = 30 m +ve


4. Substitute into y = mx + c .

C = mn + c Next Comment

becomes
C = 4n + 30 Straight Line Menu

Back to Home
Comments

• Whatever the context of the


1. Identify two points on the line. question remember:

2. Find gradient using gradient formula.


Change along y-axis
Using (0, 30) and (10, 70): Gradient =
Change along x-axis
y2 − y1 70 − 30 40
m= = = =4 • Gradient tells you rate of
x2 − x1 10 − 0 10 change so in this question:
3. Note where line cuts y-axis. Gradient = rate of hire
Cuts y-axis at (0, 30) so c = 30 40
=
10
4. Substitute into y = mx + c .
= £4 per hour
C = mn + c Next Comment

becomes
C = 4n + 30 Straight Line Menu

Back to Home
STRAIGHT LINE : Question 4B
A fuel tank holds 120 litres but develops a leak. The volume of
fuel, V litres, remaining after t hours is shown in the graph below.
Find the formula for V in terms of t.

Get hint

Reveal answer only

Go to full solution

Go to Comments

Go to Straight Line Menu

EXIT
STRAIGHT LINE : Question 4B
A fuel tank holds 120 litres but develops a leak. The volume of
fuel, V litres, remaining after t hours is shown in the graph below.
Find the formula for V in terms of t.
What would you like to do now?

Reveal answer only


1. Identify2.two
3. Note
Find
pointsgradient
onSubstitute
4. the
whereusing
(0, 120) Go to full solution
line.
intogradient
y line
= mx + c,
(NOTE
using V and t.
cuts
formula.
SCALES) vertical Go to Comments
axis
Go to Straight Line Menu

(8, 0)

EXIT
STRAIGHT LINE : Question 4B
A fuel tank holds 120 litres but develops a leak. The volume of
fuel, V litres, remaining after t hours is shown in the graph below.
Find the formula for V in terms of t.

What would you like to do now?

Go to full solution

Go to Comments

Go to Straight Line Menu

V = 120 − 15t
EXIT
Question 4B 1. Identify two points on the line.
Find the equation of this line
2. Find gradient using gradient formula.

(0, 120) Using (0, 120) and (8, 0):

y2 − y1 0 − 120 −120
m= = = = −15
x2 − x1 8−0 8
3. Note where line cuts y-axis.
(8, 0)
Cuts y-axis at (0, 120) so c = 120

4. Substitute into y = mx + c .
Begin Solution
Continue Solution V = mt + c
Comments becomes V = 120 − 15t
Straight Line Menu
What would you like to do now?
Back to Home
Comments
• Must learn gradient formula:
1. Identify two points on the line.

2. Find gradient using gradient formula.


B(x2,y2)
Using (0, 120) and (8, 0):

y2 − y1 0 − 120 −120
m= = = = −15 A(x1,y1)
x2 − x1 8−0 8
y2 – y 1
3. Note where line cuts y-axis. mAB =
x2 – x1
Cuts y-axis at (0, 120) so c = 120

4. Substitute into y = mx + c .

V = mt + c Next Comment

becomes V = 120 − 15t Straight Line Menu

Back to Home
Comments

• Must learn equation of the line:


1. Identify two points on the line.

2. Find gradient using gradient formula. Y = mx + c


Using (0, 120) and (8, 0):

y2 − y1 0 − 120 −120 gradient y - intercept


m= = = = −15
x2 − x1 8−0 8
3. Note where line cuts y-axis.

Cuts y-axis at (0, 120) so c = 120

4. Substitute into y = mx + c .

V = mt + c Next Comment

becomes V = 120 − 15t Straight Line Menu

Back to Home
Comments

• Line slopes downward from


1. Identify two points on the line. left to right

Negative gradient
2. Find gradient using gradient formula.
m -ve
Using (0, 120) and (8, 0):

y2 − y1 0 − 120 −120
m= = = = −15 • Line slopes upwards from
x2 − x1 8−0 8 left to right

3. Note where line cuts y-axis. Positive gradient

Cuts y-axis at (0, 120) so c = 120 m +ve

4. Substitute into y = mx + c .

V = mt + c Next Comment

becomes V = 120 − 15t Straight Line Menu

Back to Home
Comments

• Whatever the context of the


1. Identify two points on the line. question remember:

2. Find gradient using gradient formula.


Change along y-axis
Gradient =
Using (0, 120) and (8, 0):
Change along x-axis
y2 − y1 0 − 120 −120
m= = = = −15 • Gradient tells you rate of
x2 − x1 8−0 8 change so in this question:

3. Note where line cuts y-axis. Gradient = rate of flow


120
Cuts y-axis at (0, 120) so c = 120 = - 8
4. Substitute into y = mx + c .
= - 15 litres per hour

V = mt + c Next Comment

becomes V = 120 − 15t Straight Line Menu

Back to Home
INTERMEDIATE 2 – ADDITIONAL QUESTION BANK

You have chosen to study:

UNIT 1 :
Percentages

Please choose a question to attempt from the following:

1 2 3 4 5

Back to
EXIT Unit 1 Menu
PERCENTAGES: Question 1
(a) The speed of a car is 72mph. This falls by 12.5% as the traffic
builds up. What speed is it travelling at now?

(b) As the congestion eases the speed increases again by 6%.


Find the new speed to the nearest mph.

Get hint

Reveal answers only

Go to full solution

Go to Comments

Go to Percentages Menu

EXIT
PERCENTAGES: Question 1
(a) The speed of a car is 72mph. This falls by 12.5% as the traffic
builds up. What speed is it travelling at now?

(b) As the congestion eases the speed increases again by 6%.


Find the new speed to the nearest mph.

What would you like to do now?

Reveal answers only

1. Remember
2. For a Go to full solution
to use
decrease the
multiplication
3. For
factors factor is Go to Comments
increases
less than 1.
factor is Go to Percentages Menu
more than 1

EXIT
PERCENTAGES: Question 1
(a) The speed of a car is 72mph. This falls by 12.5% as the traffic
builds up. What speed is it travelling at now? = 63 mph
(b) As the congestion eases the speed increases again by 6%.
Find the new speed to the nearest mph. = 67 mph

What would you like to do now?

Try another like this

Go to full solution

Go to Comments

Go to Percentages Menu

EXIT
Question 1(a) 1. Find multiplier
The speed of a car is 72mph.
12.5% off leaves 87.5% or 0.875.
This falls by 12.5% as the
traffic builds up. 2. Use this multiplier to calculate new
What speed is it travelling speed
at now?
New speed = 0.875 x 72mph

= 63mph

Begin Solution
Continue Solution
Comments

Percentages Menu

Back to Home
Question 1(b) 1. Find multiplier
As the congestion eases the
6% more is 106% or 1.06.
speed increases again by 6%.
Find the new speed to the 2. Use this multiplier to calculate new
nearest mph. speed, using your answer to (a).

New speed = 1.06 x 63mph


= 66.78mph

= 67mph

Continue Solution What would you like to do now?


Try another like this
Comments

Percentages Menu

Back to Home
Comments

Percentage Decrease:
Question 1(a) 10% decrease Multiplier = 0.90
4.8% decrease Multiplier = 0.952
1. Find multiplier

12.5% off leaves 87.5% or 0.875.


Finding a Percentage:
2. Use this multiplier to calculate new
12.8% of £12500 =
speed
12.8
x 12500
New speed = 0.875 x 72mph 100

or 0.128 x 12500
= 63mph
(Preferred method)

Next Comment

Percentages Menu

Back to Home
Comments

Question 1(b) Percentage Increase:


10% increase Multiplier = 1.10
5.6% increase Multiplier = 1.056
1. Find multiplier

6% more is 106% or 1.06. Finding a Percentage:

2. Use this multiplier to calculate new 6% of 63 =

speed, using your answer to (a). 6


x 63
100
New speed = 1.06 x 63mph
or 0.06 x 63
= 66.78mph (Preferred method)

= 67mph Next Comment

Percentages Menu

Back to Home
PERCENTAGES: Question 1B
(a) The speed of a car is 60mph. This falls by 15% as the traffic
builds up. What speed is it travelling at now?

(b) As the congestion eases the speed increases again by 16%.


Find the new speed to the nearest mph.

Get hint

Reveal answers only

Go to full solution

Go to Comments

Go to Percentages Menu

EXIT
PERCENTAGES: Question 1B
(a) The speed of a car is 60mph. This falls by 15% as the traffic
builds up. What speed is it travelling at now?

(b) As the congestion eases the speed increases again by 16%.


Find the new speed to the nearest mph.

What would you like to do now?

Reveal answers only

1. Remember 2. For a Go to full solution


to usedecrease the
multiplication
factor3.isFor
factors less than 1. Go to Comments
increases
factor is Go to Percentages Menu
more than 1

EXIT
PERCENTAGES: Question 1B
(a) The speed of a car is 60mph. This falls by 15% as the traffic
builds up. What speed is it travelling at now? = 51 mph
(b) As the congestion eases the speed increases again by 16%.
Find the new speed to the nearest mph. = 59 mph

What would you like to do now?

Go to full solution

Go to Comments

Go to Percentages Menu

EXIT
Question 1B(a) 1. Find multiplier
The speed of a car is 60mph.
15% off leaves 85% or 0.85.
This falls by 15% as the
traffic builds up. 2. Use this multiplier to calculate new
What speed is it travelling speed
at now?
New speed = 0.85 x 60mph

= 51mph

Begin Solution
Continue Solution
Comments

Percentages Menu

Back to Home
Question 1B(b) 1. Find multiplier
As the congestion eases the
16% more is 116% or 1.16.
speed increases again by
16%.
2. Use this multiplier to calculate new
Find the new speed to the
speed, using your answer to (a).
nearest mph.
New speed = 1.16 x 51mph
= 59.16mph

= 59mph

Continue Solution What would you like to do now?

Comments

Percentages Menu

Back to Home
Comments

Percentage Decrease:
Question 1B(a) 10% decrease Multiplier = 0.90
4.8% decrease Multiplier = 0.952
1. Find multiplier

15% off leaves 85% or 0.85.


Finding a Percentage:
2. Use this multiplier to calculate 12.8% of £12500 =
new speed
12.8
12500
100
New speed = 0.85 x 60mph
or 0.128 x 12500
= 51mph (Preferred method)

Next Comment

Percentages Menu

Back to Home
Comments

Question 1B(b) Percentage Increase:


10% increase Multiplier = 1.10
5.6% increase Multiplier = 1.056
1. Find multiplier

16% more is 116% or 1.16. Finding a Percentage:

2. Use this multiplier to calculate 16% of 51 =


new speed, using your answer to (a).
16
x 51
100
New speed = 1.16 x 51mph
or 0.16 x 63
= 59.16mph (Preferred method)

= 59mph Next Comment

Percentages Menu

Back to Home
PERCENTAGES: Question 2
(a) The value of a car falls by 17% per annum. The car is worth £14000
when new.
Find its value after 3 years to 3 significant figures.

(b) How many more years will it take until the value is less than
half of its original value?
Get hint

Reveal answers only

Go to full solution

Go to Comments

Go to Percentages Menu

EXIT
PERCENTAGES: Question 2
(a) The value of a car falls by 17% per annum. The car is worth £14000
when new.
Find its value after 3 years to 3 significant figures.

(b) How many more years will it take until the value is less than
half of its original value?
What would you like to do now?

Reveal answers only

1.Make 2. For a
Go to full solution
repeateddecrease
use the
of factor
3. is
Keep
multiplication Go to Comments
less than 1.
multiplying
factor by factor Go to Percentages Menu
until target is
reached.
EXIT
PERCENTAGES: Question 2
(a) The value of a car falls by 17% per annum. The car is worth £14000
when new.
Find its value after 3 years to 3 significant figures. = £8010
(b) How many more years will it take until the value is less than
half of its original value? = 1 more year

What would you like to do now?

Go to full solution

Go to Comments

Go to Percentages Menu

EXIT
Question 2(a) 1. Find multiplier
The value of a car falls by
17% off leaves 83% or 0.83.
17% per annum.
The car is worth £14000 2. Use this multiplier repeatedly for
number of years indicated.
when new.
Find its value after 3 years Value after 3 years:

to 3 significant figures. = 0.83 x 0.83 x 0.83 x £14000


OR
= (0.83)3 x £14000

= £8005.018
Begin Solution
Continue Solution = £8010 to 3 sig figs.
Comments

Percentages Menu

Back to Home
Question 2(b) 1. Find target value
How many more years will
Half of original is £7000.
it take until the value is
less than half of its original 2. Keep multiplying answer to (a) until
value is below £7000.
value?
Value after 4 years = 0.83 x £8005.018
Below target = £6644.16…

Hence time required is 1 more year.

What would you like to do now?


Continue Solution
Try another like this
Comments

Percentages Menu

Back to Home
Comments
• Calculating compound
Question 2(a) Appreciation and Depreciation:

1. Find multiplier Calculation can be dealt with


year by year:
17% off leaves 83% or 0.83.
YR
0 Value = £14000
2. Use this multiplier repeatedly for
1 Value = 0.83 x 14000 = 11620
number of years indicated.
2 Value = 0.83 x 11620 = 9644.6
Value after 3 years: 3 Value = 0.83 x 9644.6 =
8005.018
= 0.83 x 0.83 x 0.83 x £14000
OR Value After 3 years = £8010
(correct to 3 sig. fig.)
= (0.83)3 x £14000
Next Comment
= £8005.018
Percentages Menu
= £8010 to 3 sig figs.
Back to Home
Comments
• Significant Figures
Question 2(a) (Large Numbers)
1. Find multiplier £4567100 rounded to 3 sig figs
17% off leaves 83% or 0.83. 3rd sig. fig.
1st sig. fig.
2. Use this multiplier repeatedly for
number of years indicated. Hence £456 [ 7100 = £4570000

Value after 3 years:


More than 5000
= 0.83 x 0.83 x 0.83 x £14000
OR Answer must be of the
same order as the
= (0.83)3 x £14000 original number
Next Comment
= £8005.018
Percentages Menu
= £8010 to 3 sig figs.
Back to Home
PERCENTAGES: Question 2B
(a) The value of a PC falls by 21% per annum. The PC costs £1200
when new.
Find its value after 4 years to 2significant figures.

(b) A company replaces its PCs when their value is less than one
quarter of the original value.
How much longer will they keep this machine?
Get hint

Reveal answers only

Go to full solution

Go to Comments

Go to Percentages Menu
EXIT
PERCENTAGES: Question 2B
(a) The value of a PC falls by 21% per annum. The PC costs £1200
when new.
Find its value after 4 years to 2significant figures.

(b) A company replaces its PCs when their value is less than one
quarter of the original value.
How much longer will they keep this machine?

What would you like to do now?

1.Make 2. For a Reveal answers only


repeateddecrease
use the
of factor
3. is
Keep Go to full solution
multiplication
less than 1.
multiplying
factor by factor Go to Comments
until target is
reached. Go to Percentages Menu
EXIT
PERCENTAGES: Question 2B
(a) The value of a PC falls by 21% per annum. The PC costs £1200
when new.
Find its value after 4 years to 2significant figures. = £470
(b) A company replaces its PCs when their value is less than one
quarter of the original value.
How much longer will they keep this machine? = 2 more years

What would you like to do now?

Go to full solution

Go to Comments

Go to Percentages Menu
EXIT
Question 2B(a) 1. Find multiplier
The value of a PC falls by
21% off leaves 79% or 0.79.
21% per annum.
The PC costs worth £1200 2. Use this multiplier repeatedly for
number of years indicated.
when new.
Find its value after 4 years Value after 4 years:

to 2 significant figures. = 0.79 x 0.79 x 0.79 x £1200


OR
= (0.79)4 x £1200

= £467.40….
Begin Solution
Continue Solution = £470 to 2 sig figs.
Comments

Percentages Menu

Back to Home
Question 2B(b) 1. Find target value
A company replaces its PCs
¼ of original value is £300.
when their value is less than
one quarter of the original 2. Keep multiplying answer to (a) until
value is below £300.
value.
How much longer will they Value after 5 years = 0.79 x £467.40

keep this machine? Above target = £369.25

Value after 6 years = 0.79 x £369.25


Below target = £291.71

Continue Solution Hence PC is kept for 2 more years.

Comments What would you like to do now?

Percentages Menu

Back to Home
Comments
• Calculating compound
Question 2B(a) Appreciation and Depreciation:

1. Find multiplier Calculation can be dealt with


year by year:
21% off leaves 79% or 0.79.
YR
2. Use this multiplier repeatedly for 0 Value = £1200
number of years indicated. 1 Value = 0.79 x 1200 = 948
2 Value = 0.79 x 948 = 748.92
Value after 4 years: 3 Value = 0.79 x 748.92 = 591.64
= 0.79 x 0.79 x 0.79 x £1200 4 Value = 0.79 x 591.64 = 467.40.
OR
Value After 4 years = £470
= (0.79) x £1200
4
(correct to 3 sig. fig.)

= £467.40…. Next Comment

Percentages Menu
= £470 to 2 sig figs.
Back to Home
Comments
• Significant Figures
Question 2B(a) (Large Numbers)
1. Find multiplier
£5632800 rounded to 3 sig figs
21% off leaves 79% or 0.79. 3rd sig. fig.
1st sig. fig.
2. Use this multiplier repeatedly for
number of years indicated. Hence £563 [ 2800 = £5630000
Value after 4 years:
Less than 5000
= 0.79 x 0.79 x 0.79 x £1200
OR Answer must be of the
same order as the
= (0.79)4 x £1200 original number
= £467.40…. Next Comment

Percentages Menu
= £470 to 2 sig figs.
Back to Home
PERCENTAGES: Question 3
Financial experts predict that house prices are about to rise by
8.5% per annum for the next few years.
A house is currently valued at £ 93000. How much is it likely to be
worth in 3 years time if this rate holds?
Give the answer to 2 significant figures!

Get hint

Reveal answers only

Go to full solution

Go to Comments

Go to Percentages Menu

EXIT
PERCENTAGES: Question 3
Financial experts predict that house prices are about to rise by
8.5% per annum for the next few years.
A house is currently valued at £ 93000. How much is it likely to be
worth in 3 years time if this rate holds?
Give the answer to 2 significant figures!

What would you like to do now?

Reveal answers only

1. Use
Go to full solution
multiplication 2. For
factor increases
repeatedly factor is Go to Comments
more than 1
Go to Percentages Menu

EXIT
PERCENTAGES: Question 3
Financial experts predict that house prices are about to rise by
8.5% per annum for the next few years.
A house is currently valued at £ 93000. How much is it likely to be
worth in 3 years time if this rate holds?
Give the answer to 2 significant figures! = £120 000

What would you like to do now?

Try another like this

Go to full solution

Go to Comments

Go to Percentages Menu

EXIT
Question 3 1. Find multiplier
Financial experts predict that
house prices are about to rise 8.5% more gives 108.5% or 1.085.
by 8.5% per annum for the
next few years. 2. Use this multiplier repeatedly for
number of years indicated.
A house is currently valued at
£ 93000. How much is it likely Value in 3 years:
to be worth in 3 years time if
this rate holds? = 1.085 x 1.085 x 1.085 x £93000
Give the answer to 2 OR
significant figures!
= (1.085)3 x £93000

= £118787.88
Continue Solution
Try another like this = £120 000 to 2 sig figs.
Comments
What would you like to do now?
Percentages Menu

Back to Home
Comments
• Calculating compound
Appreciation and Depreciation:
1. Find multiplier
Calculation can be dealt with
8.5% more gives 108.5% or 1.085. year by year:

2. Use this multiplier repeatedly for YR


number of years indicated. 0 Value = £93 000
1 Value = 1.085 x 93000 = 100905
Value in 3 years: 2 Value = 1.085 x 100905=
109481.92…
= 1.085 x 1.085 x 1.085 x £93000
3 Value = 1.085 x 1909481.92.. =
OR 118787.88
= (1.085)3 x £93000
Value After 3 years = £120 000
= £118787.88
(correct to 2 sig. fig.)
Next Comment
= £120 000 to 2 sig figs. Percentages Menu

Back to Home
Comments
• Significant Figures

1. Find multiplier (Large Numbers)

8.5% more gives 108.5% or 1.085. £118787.88 rounded to 2 sig figs

2nd sig. fig.


2. Use this multiplier repeatedly for 1st sig. fig.
number of years indicated.
Hence £11 [ 8787.88 = £120 000
Value in 3 years:
= 1.085 x 1.085 x 1.085 x £93000
More than 5000
OR
Answer must be of the
= (1.085)3 x £93000
same order as the
original number
= £118787.88
Next Comment
= £120 000 to 2 sig figs. Percentages Menu

Back to Home
PERCENTAGES: Question 3B
It has been predicted that the population of a town
will rise by 7.3% per annum for the next few years.
The population is currently 32000. How big will
this become in 4 years time if this rate stays the same?
Give the answer to 3 significant figures!
Get hint

Reveal answers only

Go to full solution

Go to Comments

Go to Percentages Menu

EXIT
PERCENTAGES: Question 3B
It has been predicted that the population of a town
will rise by 7.3% per annum for the next few years.
The population is currently 32000. How big will
this become in 4 years time if this rate stays the same?
Give the answer to 3 significant figures!

What would you like to do now?

Reveal answers only

1. Use
Go to full solution
multiplication 2. For
factor increases
repeatedly factor is Go to Comments
more than 1
Go to Percentages Menu

EXIT
PERCENTAGES: Question 3B
It has been predicted that the population of a town
will rise by 7.3% per annum for the next few years.
The population is currently 32000. How big will
this become in 4 years time if this rate stays the same?
Give the answer to 3 significant figures! = 42 400

What would you like to do now?

Go to full solution

Go to Comments

Go to Percentages Menu

EXIT
Question 3B 1. Find multiplier
It has been predicted that the
7.3% more gives 107.3% or 1.073.
population of a town will rise
by 7.3% per annum for the
next few years. 2. Use this multiplier repeatedly for
number of years indicated.
The population is currently
32000. How big will this Value in 4 years:
become in 4 years time if this
rate stays the same? = 1.073 x 1.073 x 1.073 x 1.073 x 32000

Give the answer to 3 OR


significant figures! = (1.073)4 x 32000

= 42 417.8…
Continue Solution

= 42 400 to 3 sig figs.


Comments
What would you like to do now?
Percentages Menu

Back to Home
Comments
• Calculating compound
Appreciation and Depreciation:
1. Find multiplier
Calculation can be dealt with
year by year:
7.3% more gives 107.3% or 1.073.
YR
0 Value = £32 000
2. Use this multiplier repeatedly for 1 Value = 1.073 x 32000 = 34336
number of years indicated. 2 Value = 1.073 x 34336 =
36842.52..
Value in 4 years: 3 Value = 1.073 x 36842.52.. =
= 1.073 x 1.073 x 1.073 x 1.073x32000 39532.03..
4 Value = 1.073 x 39532.03.. =
OR 42417.87
= (1.073)4 x 32000
Value after 4 years = £42 400
= 42 417.8… (correct to 3 sig. fig.)
Next Comment
= 42 400 to 3 sig figs. Percentages Menu

Back to Home
Comments
• Significant Figures

1. Find multiplier (Large Numbers)

7.3% more gives 107.3% or 1.073. £118787.88 rounded to 2 sig figs

2nd sig. fig.


2. Use this multiplier repeatedly for 1st sig. fig.
number of years indicated.
Hence £11 [ 8787.88 = £120 000
Value in 4 years:
= 1.073 x 1.073 x 1.073 x 1.073 x 32000
More than 5000
OR
Answer must be of the
= (1.073)4 x 32000
same order as the
original number
= 42 417.8…
Next Comment
= 42 400 to 3 sig figs. Percentages Menu

Back to Home
PERCENTAGES: Question 4
A police force announced that its annual crime figures were 11130.
This was a 16% fall on the previous year’s figure.
What would the previous year’s figure have been?

Get hint

Reveal answers only

Go to full solution

Go to Comments

Go to Percentages Menu

EXIT
PERCENTAGES: Question 4
A police force announced that its annual crime figures were 11130.
This was a 16% fall on the previous year’s figure.
What would the previous year’s figure have been?

What would you like to do now?

Reveal answers only

1. For a 2. Express Go to full solution


decrease new
the figure in
factor is less
terms of old
3. Substitute
than 1. Go to Comments
using in this
known
factor.
value and Go to Percentages Menu
solve.

EXIT
PERCENTAGES: Question 4
A police force announced that its annual crime figures were 11130.
This was a 16% fall on the previous year’s figure.
What would the previous year’s figure have been? = 13 250

What would you like to do now?

Try another like this

Go to full solution

Go to Comments

Go to Percentages Menu

EXIT
Question 4 1. Find multiplier
A police force announced
that its annual crime figures 16% fall gives 84% or 0.84.
were 11130.
2. Using this multiplier, express new
This was a 16% fall on the
figure in terms of old figure.
previous year’s figure.
What would the previous New figure = 0.84 x old figure
year’s figure have been?
3. Substitute known value and solve.

Old figure = 11130 ÷ 0.84


= 13250
Continue Solution Previous figure was 13250
Try another like this
Comments What would you like to do now?
Percentages Menu

Back to Home
Comments

Alternative approach:
1. Find multiplier
After a 16% reduction
16% fall gives 84% or 0.84.
84% represents 11130
Require 100%
2. Using this multiplier, express new
figure in terms of old figure. Using simple proportion:
New figure = 0.84 x old figure
84% 11130
3. Substitute known value and 1% 11130 ÷ 84
solve. 100% 11130 ÷ 84 x 100
Old figure = 11130 ÷ 0.84 = 13250
= 13250

Previous figure was 13250 Next Comment

Percentages Menu

Back to Home
PERCENTAGES: Question 4B

After receiving a 6.5% pay rise a nurse now earns £15336 per
annum.
What was her previous income?

Get hint

Reveal answers only

Go to full solution

Go to Comments

Go to Percentages Menu

EXIT
PERCENTAGES: Question 4B

After receiving a 6.5% pay rise a nurse now earns £15336 per
annum.
What was her previous income?

What would you like to do now?

Reveal answers only

1. For an 2. Express
Go to full solution
increase the
new figure in
factor is more
terms of old
3. Substitute
than 1. Go to Comments
usingin this
known
factor.
value and Go to Percentages Menu
solve.

EXIT
PERCENTAGES: Question 4B

After receiving a 6.5% pay rise a nurse now earns £15336 per
annum.
What was her previous income? = £14 400

What would you like to do now?

Go to full solution

Go to Comments

Go to Percentages Menu

EXIT
Question 4B 1. Find multiplier
After receiving a 6.5% pay 6.5% increase gives 106.5% or 1.065.
rise a nurse now earns
£15336 per annum.
2. Using this multiplier, express new
What was her previous figure in terms of old figure.
income?
New figure = 1.065 x old figure
3. Substitute known value and solve.

Old figure = £15 336 ÷ 1.065


= £14 400
Continue Solution Previous income was £14 400

Comments What would you like to do now?


Percentages Menu

Back to Home
Comments

Alternative approach:

1. Find multiplier After a 6.5% increase


6.5% increase gives 106.5% or 1.065. 106.5% represents 15336
Require 100%
2. Using this multiplier, express new
figure in terms of old figure. Using simple proportion:

New figure = 1.065 x old figure 106.5% 15336


3. Substitute known value and solve. 1% 15336 ÷ 106.5
100% 15336 ÷ 106.5 x 100
= 14 400
Old figure = £15 336 ÷ 1.065
= £14 400

Previous income was £14 400 Next Comment

Percentages Menu

Back to Home
PERCENTAGES: Question 5
Between 2002 & 2003 the grain production for a particular country
rose from 4.3million tonnes to 5.1million tonnes.
What percentage increase is this?

Get hint

Reveal answers only

Go to full solution

Go to Comments

Go to Percentages Menu

EXIT
PERCENTAGES: Question 5
Between 2002 & 2003 the grain production for a particular country
rose from 4.3million tonnes to 5.1million tonnes.
What percentage increase is this?

What would you like to do now?

Reveal answers only

1. Find the
2. Make this a Go to full solution
actual
fraction of the
increase.
original and Go to Comments
convert to a
decimal. Go to Percentages Menu

EXIT
PERCENTAGES: Question 5
Between 2002 & 2003 the grain production for a particular country
rose from 4.3million tonnes to 5.1million tonnes.
What percentage increase is this? = 18.6%

What would you like to do now?


Try another like this

Go to full solution

Go to Comments

Go to Percentages Menu

EXIT
Question 5 1. Find actual increase

Between 2002 & 2003 the 5.1m – 4.3m = 0.8m


grain production for a
2. Make this a fraction of the original
particular country rose from and convert to a decimal.
4.3million tonnes to 0.8
/4.3 = 0.8 ÷ 4.3 = 0.1860…
5.1million tonnes.
= 0.186 to 3 sig figs
What percentage increase is
this? 3. Multiply by 100 to change to %.

0.186 x 100 = 18.6


Continue Solution
So % increase is 18.6%
Try another like this
Comments What would you like to do now?
Percentages Menu

Back to Home
Comments
Using formula:

1. Find actual increase to express a as a percentage of b:

5.1m – 4.3m = 0.8m


a x 100%
b
2. Make this a fraction of the original
and convert to a decimal.
So to express an increase of
0.8
/4.3 = 0.8 ÷ 4.3 = 0.1860… 0.8 as a percentage of 4.3:
= 0.186 to 3 sig figs 0.8 x 100%
=
4.3
3. Multiply by 100 to change to %.

0.186 x 100 = 18.6 = 18.6% (1 dec. pl.)


Try Another
So % increase is 18.6%
Percentages Menu

Back to Home
PERCENTAGES: Question 5B

Between 2000 & 2001 the number of customers served by a power


company fell from 5.1million to 4.3million.
What percentage decrease is this?

Get hint

Reveal answers only

Go to full solution

Go to Comments

Go to Percentages Menu

EXIT
PERCENTAGES: Question 5B

Between 2000 & 2001 the number of customers served by a power


company fell from 5.1million to 4.3million.
What percentage decrease is this?

What would you like to do now?

Reveal answers only

1. Find the
2. Make this a Go to full solution
actual
fraction of the
decrease.
original and Go to Comments
convert to a
decimal. Go to Percentages Menu

EXIT
PERCENTAGES: Question 5B

Between 2000 & 2001 the number of customers served by a power


company fell from 5.1million to 4.3million.
What percentage decrease is this? = 15.7%

What would you like to do now?

Go to full solution

Go to Comments

Go to Percentages Menu

EXIT
Question 5B 1. Find actual decrease
Between 2000 & 2001 the 5.1m – 4.3m = 0.8m
number of customers
served by a power company
fell from 5.1million to 2. Make this a fraction of the original
4.3million. and convert to a decimal.

What percentage decrease


0.8
/5.1 = 0.8 ÷ 5.1 = 0.1568…
is this? = 0.157 to 3 sig figs

3. Multiply by 100 to change to %.

0.157 x 100 = 15.7


Continue Solution
So % increase is 15.7%

Comments What would you like to do now?


Percentages Menu

Back to Home
Comments
Using formula:

1. Find actual decrease to express a as a percentage of b:

5.1m – 4.3m = 0.8m


a x 100%
b
2. Make this a fraction of the original
and convert to a decimal.
So to express a decrease of
0.8
/5.1 = 0.8 ÷ 5.1 = 0.1568… 0.8 as a percentage of 5.1:
= 0.157 to 3 sig figs 0.8 x 100%
=
5.1
3. Multiply by 100 to change to %.

0.157 x 100 = 15.7 = 15.7% (1 dec. pl.)


Next Comment
So % increase is 15.7%
Percentages Menu

Back to Home
INTERMEDIATE 2 – ADDITIONAL QUESTION BANK

You have chosen to study:

UNIT 1 :
Volumes

Please choose a question to attempt from the following:

1 2 3 4 5 6

Back to
EXIT Unit 1 Menu
VOLUMES: Question 1
The dome on a cathedral is a hemisphere of diameter 25 metres.
Find the volume of this dome correct to 2 significant figures.

Get hint

Reveal answers only

Go to full solution

Go to Comments

Go to Volumes Menu

EXIT
VOLUMES: Question 1
The dome on a cathedral is a hemisphere of diameter 25 metres.
Find the volume of this dome correct to 2 significant figures.

V = 43 π r 3

What would you like to do now?

Reveal answers only

1. Volume of 2. For
Go to full solution
sphere: hemisphere
V = 43 π rdivide
3 3. Remember
by 2.
Go to Comments
to use
RADIUS. Go to Volumes Menu

EXIT
VOLUMES: Question 1
The dome on a cathedral is a hemisphere of diameter 25 metres.
Find the volume of this dome correct to 2 significant figures.

= 4100 m3

What would you like to do now?

Try another like this

Go to full solution

Go to Comments

Go to Volumes Menu

EXIT
Question 1 1. Make sure you use radius.
The dome on a cathedral d = 25 m so r = 12.5m
is a hemisphere of diameter
2. Use formula to find volume.
25 metres.
Find the volume of this V = π r = 4 ÷ 3 × π × (12.5)3
4
3
3

dome correct to 2
= 8181.23…m3
significant figures.
3. Divide by 2 for hemisphere

Volume of hemisphere = 8181 ⋅ 2 ÷ 2


Continue Solution
= 4090.6…m3

Try another like this So Volume is 4100 m3


Comments
What would you like to do now?
Volumes Menu

Back to Home
Comments

Refer to formula sheet :


1. Make sure you use radius. 4 3
Volume of a Sphere = πr
3
d = 25 m so r = 12.5m
r = radius
2. Use formula to find volume.

V = π r = 4 ÷ 3 × π × (12.5)3
4
3
3
As hemisphere is half a sphere:
2 3
= 8181.23…m3 Volume of a Hemisphere = πr
3
3. Divide by 2 for hemisphere r = radius

Volume of hemisphere = 8181 ⋅ 2 ÷ 2


= 4090.6…m3
Next Comment
So Volume is 4100 m3 Volumes Menu

Back to Home
Comments

Units of Volume:
1. Make sure you use radius.
cubic centimetres cm3
d = 25 m so r = 12.5m
cubic metres m3
2. Use formula to find volume.
For cylinders and spheres
V = π r = 4 ÷ 3 × π × (12.5)
4
3
3 3
make sure you identify
the radius.
= 8181.23…m3 Often exam questions
give dimensions in terms of the
3. Divide by 2 for hemisphere
diameter.
Volume of hemisphere = 8181 ⋅ 2 ÷ 2
= 4090.6…m3
Next Comment
So Volume is 4100 m3 Volumes Menu

Back to Home
VOLUMES: Question 1B
An igloo has a diameter of 2.8metres.
Find its volume correct to 2 significant figures.

Get hint

Reveal answers only

Go to full solution

Go to Comments

Go to Volumes Menu

EXIT
VOLUMES: Question 1B
An igloo has a diameter of 2.8metres.
Find its volume correct to 2 significant figures.

V = 43 π r 3

What would you like to do now?

Reveal answers only

1. Volume of 2. For
Go to full solution
sphere: hemisphere
V = 43 π rdivide
3 3. Remember
by 2.
Go to Comments
to use
RADIUS. Go to Volumes Menu

EXIT
VOLUMES: Question 1B
An igloo has a diameter of 2.8metres.
Find its volume correct to 2 significant figures. = 5.7 m3

What would you like to do now?

Go to full solution

Go to Comments

Go to Volumes Menu

EXIT
Question 1B 1. Make sure you use radius.

An igloo has a diameter of d = 2.8 m so r = 1.4m

2.8metres.
2. Use formula to find volume.
Find its volume correct to 2
significant figures.
V = π r = 4 ÷ 3 × π × (1 ⋅ 4)3
4
3
3

= 11.49…m3

3. Divide by 2 for hemisphere

Volume of hemisphere = 11 ⋅ 49 ÷ 2
Continue Solution
= 5.747…m3
So Volume is 5.7 m3
Comments
What would you like to do now?
Volumes Menu

Back to Home
Comments

Refer to formula sheet :


1. Make sure you use radius. 4 3
Volume of a Sphere = πr
3
d = 2.8 m so r = 1.4m
r = radius
2. Use formula to find volume.

V = π r = 4 ÷ 3 × π × (1 ⋅ 4)3
4
3
3
As hemisphere is half a sphere:
2 3
= 11.49…m3 Volume of a Hemisphere = πr
3
3. Divide by 2 for hemisphere r = radius

Volume of hemisphere = 11 ⋅ 49 ÷ 2
= 5.747…m3
Next Comment
So Volume is 5.7 m3 Volumes Menu

Back to Home
Comments

Units of Volume:
1. Make sure you use radius.
cubic centimetres cm3
d = 2.8 m so r = 1.4m
cubic metres m3
2. Use formula to find volume.
For cylinders, cones and
V = π r = 4 ÷ 3 × π × (1 ⋅ 4)
4
3
3 3
spheres make sure you identify
the radius.
= 11.49…m3 Often exam questions
give dimensions in terms of the
3. Divide by 2 for hemisphere
diameter.
Volume of hemisphere = 11 ⋅ 49 ÷ 2
= 5.747…m3
Next Comment
So Volume is 5.7 m3 Volumes Menu

Back to Home
VOLUMES: Question 2 part (a)
The shelter on a station platform is 4m long. Its cross section consists
of ¼ circle above a rectangle which is 2m high by 1m wide as shown.
Find its volume correct to 3 significant figures.

GO TO
PART (b)
2m

4m
1m

EXIT
VOLUMES: Question 2 part (b)
The shelter is modified so that the roof has a straight slope making
the cross-section a trapezium which is 2.5m high at the back and 2m
high at the front.
If the volume is unchanged then how wide will this new design be?

Get hint

Reveal answers only

Go to full solution
2.5 2
m m Go to Comments

Go to Volumes Menu
4 W
m m BACK TO
PART (a)
EXIT
VOLUMES: Question 2 part (a)
The shelter on a station platform is 4m long. Its cross section consists
of ¼ circle above a rectangle which is 2m high by 1m wide as shown.
Find its volume correct to 3 significant figures.

What would you like to do now?


Get hint for part (b)

Reveal answers only


1. Break base/
end into basic
Go to full solution
parts. Find area
of each part2.then
Do not 2m
Go to Comments
add for total round
area
3. Volume
of base. answers until =
Area
4m of base
end. Go to Volumes Menu
x length
1m

EXIT
VOLUMES: Question 2 part (b)
The shelter is modified so that the roof has a straight slope making
the cross-section a trapezium which is 2.5m high at the back and 2m
high at the front.
If the volume is unchanged then how wide will this new design be?

What would you like to do now?

Reveal answers
1. Length is
unchanged so
Go to full solution
only need to
2. Find 2.5 2
compare areas.
expression for m m Go to Comments
total 3.
area in
Equate
terms this
of W.to Go to Volumes Menu
4
previous W
m and
area m
solve for W.
EXIT
VOLUMES: Question 2 part (a)
The shelter on a station platform is 4m long. Its cross section consists
of ¼ circle above a rectangle which is 2m high by 1m wide as shown.
Find its volume correct to 3 significant figures.
= 11.1 m3
What would you like to do now?

Reveal answer to (b)

Go to full solution

2m
Go to Comments

4m Go to Volumes Menu

1m

EXIT
VOLUMES: Question 2 part (b)
The shelter is modified so that the roof has a straight slope making
the cross-section a trapezium which is 2.5m high at the back and 2m
high at the front. = 4.93 m3
If the volume is unchanged then how wide will this new design be?

What would you like to do now?

Try another like this

Go to full solution
2.5 2
m m Go to Comments

Go to Volumes Menu
4 W
m m

EXIT
Question 2 part (a) 1. Find total area of base
The shelter on a station
platform is 4m long. Its Area of rectangle = 2 x 1 = 2 m2
cross section consists of ¼ Area ¼ circle = π r2 ÷ 4
= π x 1 x 1÷
circle above a rectangle 4
which is 2m high by 1m = 0.785…m2
wide as shown. Total area = 2 + 0.785 = 2.785..m3
Find its volume correct to 3 2. Find volume by multiplying base
significant figures. area by length.

2 Volume = 2.785…x 4
m
4 1 = 11.14….m3
m m = 11.1m3 to 3sfs
Continue Solution
Try another like this So Volume is 11.1 m3
Comments

Volumes Menu

Back to Home
Question 2 part (b) 1. Find total area of base
The shelter is modified so that
the roof has a straight slope Area of rectangle = 2 x W = 2W m2
making the cross-section a Area of triangle = ½ x W x 0.5
trapezium which is 2.5m high
at the back and 2m high at the = 0.25W m2
front. Total area = 2W + 0.25W = 2.25Wm3
If the volume is unchanged 2. As length unchanged previous
then how wide will this new base area and this base area must be
design be? same if volume remains unchanged.
2 2 So 2.25 W = 11.1
. m
5 W Then W = 11.1 ÷ 2.25
m = 4.93333…
Continue Solution
m
Try another like this So width is 4.93 m
Comments
What would you like to do now?
Volumes Menu

Back to Home
Comments

Formulae which are not on the


Question 2 part (a) given formulae sheet:
1. Find total area of base
Area of rectangle = 2 x 1 = 2 m2
Area of circle =πr 2
r

Area ¼ circle = π r2 ÷ 4
= π x 1 x 1÷
4
= 0.785…m2
Total area = 2 + 0.785 = 2.785..m3 Area of rectangle = l x b
l
2. Find volume by multiplying base
area by length. b
Volume = 2.785…x 4
= 11.14….m3
= 11.1m3 to 3sfs Next Comment

So Volume is 11.1 m3 Volumes Menu

Back to Home
Comments
Split base into two shapes:
Question 2 part (a)
Rectangle and quarter circle.
1. Find total area of base
Area of rectangle = 2 x 1 = 2 m2
Area ¼ circle = π r2 ÷ 4
= π x 1 x 1÷
4
= 0.785…m2
Total area = 2 + 0.785 = 2.785..m3
2 2m
2. Find volume by multiplying base m
area by length.

Volume = 2.785…x 4 1 1m
m
= 11.14….m3
= 11.1m3 to 3sfs Next Comment

So Volume is 11.1 m3 Volumes Menu

Back to Home
Comments
Split base into two shapes:
Question 2 part (a)
Rectangle and quarter circle.
1. Find total area of base
Area of rectangle = 2 x 1 = 2 m2
Area ¼ circle = π r2 ÷ 4
= π x 1 x 1÷
4
= 0.785…m2
1m
Total area = 2 + 0.785 = 2.785..m3
2
2. Find volume by multiplying base m
area by length.

Volume = 2.785…x 4 1
m
= 11.14….m3
= 11.1m3 to 3sfs Next Comment

So Volume is 11.1 m3 Volumes Menu

Back to Home
Comments

Areas of PART of circles can be


Question 2 part (a) found by calculating full area and
1. Find total area of base then:

Area of rectangle = 2 x 1 = 2 m2 dividing by 2 for semi-circle


Area ¼ circle = π r2 ÷ 4
= π x 1 x 1÷
dividing by 4 for quarter-circle
4
= 0.785…m2
Total area = 2 + 0.785 = 2.785..m3
2. Find volume by multiplying base
area by length.

Volume = 2.785…x 4
= 11.14….m3
= 11.1m3 to 3sfs Next Comment

So Volume is 11.1 m3 Volumes Menu

Back to Home
Comments

Remember you must find TOTAL


Question 2 part (a) area of base.
1. Find total area of base
Area of rectangle = 2 x 1 = 2 m2
Area ¼ circle = π r2 ÷ 4
= π x 1 x 1÷
4
= 0.785…m2
Total area = 2 + 0.785 = 2.785..m3 Volume of prism
2. Find volume by multiplying base = area of base x height
area by length.

Volume = 2.785…x 4
= 11.14….m3
= 11.1m3 to 3sfs Next Comment

So Volume is 11.1 m3 Volumes Menu

Back to Home
Comments

Question 2 part (b) As shapes composing base have


changed we must recalculate
area of base.
1. Find total area of base
Area of rectangle = 2 x W = 2W m2 height
Area of triangle = ½ x W x 0.5
= 0.25W m2
base
Total area = 2W + 0.25W = 2.25Wm3
Area of triangle =
2. As length unchanged previous
base area and this base area must be ½ x base x height
same if volume remains unchanged.
So 2.25 W = 11.1 OR (base x height) ÷2
Then W = 11.1 ÷ 2.25
Next Comment
= 4.93333…
Volumes Menu
So width is 4.93 m
Back to Home
Comments

Question 2 part (b)


You must now find the width
1. Find total area of base which when multiplied by this
new base area will give us the
Area of rectangle = 2 x W = 2W m2 volume found in part (a) of the
Area of triangle = ½ x W x 0.5 question.

= 0.25W m2
Total area = 2W + 0.25W = 2.25Wm3
2. As length unchanged previous
base area and this base area must be
same if volume remains unchanged.
So 2.25 W = 11.1
Then W = 11.1 ÷ 2.25
Next Comment
= 4.93333…
Volumes Menu
So width is 4.93 m
Back to Home
VOLUMES: Question 2B part (a)

A bus shelter is 3m long. Its cross section consists of ¼ circle above


a rectangle which is 2m high by 1.2m wide as shown.
Find its volume correct to 3 significant figures.

GO TO
PART (b)
2m

3m
1.2m

EXIT
VOLUMES: Question 2B part (b)
The shelter is modified so that the roof has a straight slope making the
cross-section a trapezium which is 2.4m high at the back and 2m high
at the front.
If the volume is unchanged then how wide will this new design be?

Get hint

Reveal answers only

Go to full solution
2.4 2
m Go to Comments
m
Go to Volumes Menu
3 W
m m BACK TO
PART (a)
EXIT
VOLUMES: Question 2B part (a)
A bus shelter is 3m long. Its cross section consists of ¼ circle above
a rectangle which is 2m high by 1.2m wide as shown.
Find its volume correct to 3 significant figures.

What would you like to do now?


Get hint for part (b)

Reveal answers only


1. Break base/
end into basic
Go to full solution
parts. Find area
2. Do not
of each part then 2m Go to Comments
round
add for total area
3. Volume
of base. answers until=
Area
end.of base Go to Volumes Menu
x length
3m
1.2m

EXIT
VOLUMES: Question 2B part (b)
The shelter is modified so that the roof has a straight slope making the
cross-section a trapezium which is 2.4m high at the back and 2m high
at the front.
If the volume is unchanged then how wide will this new design be?

What would you like to do now?

Reveal answers
1. Length is
unchanged so
Go to full solution
only need to
2. Find
compare areas. 2.4 2
expression for m Go to Comments
m
total 3.
area in
Equate
terms this
of W.to Go to Volumes Menu
3
previous W
m and
area m
solve for W.
EXIT
VOLUMES: Question 2B part (a)
A bus shelter is 3m long. Its cross section consists of ¼ circle above
a rectangle which is 2m high by 1.2m wide as shown.
Find its volume correct to 3 significant figures.
= 10.6 m3
What would you like to do now?

Reveal answer to (b)

Go to full solution

2m Go to Comments

Go to Volumes Menu
3m
1.2m

EXIT
VOLUMES: Question 2B part (b)
The shelter is modified so that the roof has a straight slope making the
cross-section a trapezium which is 2.4m high at the back and 2m high
at the front. = 4.82 m3
If the volume is unchanged then how wide will this new design be?

What would you like to do now?

Go to full solution
2.4 2
m Go to Comments
m
Go to Volumes Menu
3 W
m m
EXIT
Question 2B part (a) 1. Find total area of base
A bus shelter is 3m long. Its
cross section consists of ¼ Area of rectangle = 2 x 1.2 = 2.4 m2
circle above a rectangle Area ¼ circle = π r2 ÷ 4
= π x 1.2 x 1.2 ÷
which is 2m high by 1.2m 4
wide as shown. = 1.130…m2

Find its volume correct to 3 Total area = 2.4 + 1.130.. = 3.530..m3


significant figures. 2. Find volume by multiplying base
area by length.

2 Volume = 3.530…x 3
m
3 = 10.592….m3
m 1.
2 = 10.6m3 to 3sfs
m
Continue Solution So Volume is 10.6 m3
Comments

Volumes Menu

Back to Home
Question 2B part (b) 1. Find total area of base
The shelter is modified so that
the roof has a straight slope Area of rectangle = 2 x W = 2W m2
making the cross-section a Area of triangle = ½ x W x 0.4
trapezium which is 2.4m high
at the back and 2m high at the = 0.2W m2
front. Total area = 2W + 0.2W = 2.2Wm3
If the volume is unchanged 2. As length unchanged previous
then how wide will this new base area and this base area must be
design be? same if volume remains unchanged.
2 2 So 2.2 W = 10.6
. m
4 W Then W = 10.6 ÷ 2.2
m = 4.8181…
m
Continue Solution So width is 4.82 m
Comments
What would you like to do now?
Volumes Menu

Back to Home
Comments

Formulae which are not on the


Question 2B part (a) given formulae sheet:

=πr 2
1. Find total area of base
Area of circle r
Area of rectangle = 2 x 1.2 = 2.4 m2
Area ¼ circle = π r2 ÷ 4
= π x 1.2 x 1.2 ÷
4
= 1.130…m2
Area of rectangle = l x b
Total area = 2.4 + 1.130.. = 3.530..m 3
l
2. Find volume by multiplying base
area by length. b

Volume = 3.530…x 3
= 10.592….m3
Next Comment
= 10.6m3 to 3sfs
Volumes Menu
So Volume is 10.6 m 3

Back to Home
Comments
Split base into two shapes:
Question 2B part (a)
Rectangle and quarter circle.
1. Find total area of base
Area of rectangle = 2 x 1.2 = 2.4 m2
Area ¼ circle = π r2 ÷ 4
= π x 1.2 x 1.2 ÷
4
= 1.130…m2
Total area = 2.4 + 1.130.. = 3.530..m3 2 2m
m
2. Find volume by multiplying base
area by length.

Volume = 3.530…x 3 1. 1.2 m


2
= 10.592….m3 m
Next Comment
= 10.6m3 to 3sfs
Volumes Menu
So Volume is 10.6 m 3

Back to Home
Comments
Split base into two shapes:
Question 2B part (a)
Rectangle and quarter circle.
1. Find total area of base
Area of rectangle = 2 x 1.2 = 2.4 m2
Area ¼ circle = π r2 ÷ 4
= π x 1.2 x 1.2 ÷
4
= 1.130…m2
1.2 m
Total area = 2.4 + 1.130.. = 3.530..m3 2
m
2. Find volume by multiplying base
area by length.

Volume = 3.530…x 3 1.
2
= 10.592….m3 m
Next Comment
= 10.6m3 to 3sfs
Volumes Menu
So Volume is 10.6 m 3

Back to Home
Comments

Areas of PART of circles can be


Question 2B part (a) found by calculating full area and
then:
1. Find total area of base
dividing by 2 for semi-circle
Area of rectangle = 2 x 1.2 = 2.4 m2
Area ¼ circle = π r2 ÷ 4
= π x 1.2 x 1.2 ÷ dividing by 4 for quarter-circle
4
= 1.130…m2
Total area = 2.4 + 1.130.. = 3.530..m3
2. Find volume by multiplying base
area by length.

Volume = 3.530…x 3
= 10.592….m3
Next Comment
= 10.6m3 to 3sfs
Volumes Menu
So Volume is 10.6 m 3

Back to Home
Comments

Remember you must find TOTAL


Question 2B part (a) area of base.

1. Find total area of base


Area of rectangle = 2 x 1.2 = 2.4 m2
Area ¼ circle = π r2 ÷ 4
= π x 1.2 x 1.2 ÷
4
= 1.130…m2
Total area = 2.4 + 1.130.. = 3.530..m3 Volume of prism
2. Find volume by multiplying base = area of base x height
area by length.

Volume = 3.530…x 3
= 10.592….m3
Next Comment
= 10.6m3 to 3sfs
Volumes Menu
So Volume is 10.6 m 3

Back to Home
Comments

As shapes composing base have


Question 2B part (b) changed we must recalculate
area of base.
1. Find total area of base
Area of rectangle = 2 x W = 2W m2 height
Area of triangle = ½ x W x 0.4
= 0.2W m2
base
Total area = 2W + 0.2W = 2.2Wm3
Area of triangle =
2. As length unchanged previous
base area and this base area must be ½ x base x height
same if volume remains unchanged.
So 2.2 W = 10.6 OR (base x height) ÷2
Then W = 10.6 ÷ 2.2
Next Comment
= 4.8181…
Volumes Menu
So width is 4.82 m
Back to Home
Comments

Question 2B part (b)


You must now find the width
1. Find total area of base which when multiplied by this
new base area will give us the
Area of rectangle = 2 x W = 2W m2 volume found in part (a) of the
Area of triangle = ½ x W x 0.4 question.

= 0.2W m2
Total area = 2W + 0.2W = 2.2Wm3
2. As length unchanged previous
base area and this base area must be
same if volume remains unchanged.
So 2.2 W = 10.6
Then W = 10.6 ÷ 2.2
Next Comment
= 4.8181…
Volumes Menu
So width is 4.82 m
Back to Home
VOLUMES: Question 3
The turret on a castle consists of a cylindrical tower of diameter 3m and
height 7m.
Its conical roof is 2.5m high.

Find the volume of the turret to 2 significant figures.

Get hint

Reveal answers only


turret
Go to full solution

Go to Comments

Go to Volumes Menu

EXIT
VOLUMES: Question 3
The turret on a castle consists of a cylindrical tower of diameter 3m and
height 7m. Its conical roof is 2.5m high.

Find the volume of the turret to 2 significant figures.


Vcone V
= π r h cylinder
1 2 = π r 2
h
3

What would you like to do now?

Reveal answers only


turret
1. Volume2. of
Volume of
Go to full solution
cylinder: cone:
V π
2 =3.1Remember
V = π r h 3 to use r 2
h Go to Comments
RADIUS.
Go to Volumes Menu

EXIT
VOLUMES: Question 3
The turret on a castle consists of a tower of diameter 3m and height 7m.
Its conical roof is 2.5m high.

Find the volume of the turret to 2 significant figures. = 55 m3

What would you like to do now?

Try another like this


turret
Go to full solution

Go to Comments

Go to Volumes Menu

EXIT
Question 3 1. Make sure you use radius.
The turret on a castle
consists of a cylindrical d = 3 m so r = 1.5m
ower of diameter 3m and
height 7m. Its conical roof is 2. Use formulae to find volume of
2.5m high. each part of structure .
Find the volume of the Vol tower = π r2h = π ×1.5 × 7
urret to 2 significant figures. = 49.48…m3
Vol roof = 1/3π r2h= π × (1.5) 2 × 2.5 ÷ 3
= 5.89…m3
3. Add to find total volume.
Total Volume = 49.48.. + 5.89..
Continue Solution
= 55.37…m3
Try another like this
Comments So Volume is 55 m3

Volumes Menu
What would you like to do now?

Back to Home
Comments

Refer to formula sheet :


1. Make sure you use radius.

d = 3 m so r = 1.5m Vcone = 13 π r 2 h

2. Use formulae to find volume of


each part of structure .
Vol tower = π r2h = π ×1.5 × 7 height h
= 49.48…m3
Vol roof = 1/3π r2h= π × (1.5) 2 × 2.5 ÷ 3
= 5.89…m3
radius r
3. Add to find total volume.
Total Volume = 49.48.. + 5.89..
Next Comment
= 55.37…m 3

Volumes Menu
So Volume is 55 m3
Back to Home
Comments

Refer to formula sheet :


1. Make sure you use radius.

d = 3 m so r = 1.5m
Vcylinder = π r 2 h

2. Use formulae to find volume of radius r


each part of structure .
Vol tower = π r2h = π ×1.5 × 7
= 49.48…m3
height h
Vol roof = /3π r h= π × (1.5) × 2.5 ÷ 3
1 2
2

= 5.89…m3
3. Add to find total volume.
Total Volume = 49.48.. + 5.89..
Next Comment
= 55.37…m3

Volumes Menu
So Volume is 55 m3
Back to Home
Comments

For cylinders, cones and


1. Make sure you use radius. spheres make sure you identify
the radius.
d = 3 m so r = 1.5m Often exam questions
give dimensions in terms of the
2. Use formulae to find volume of diameter.
each part of structure .
Vol tower = π r2h = π ×1.5 × 7 Make sure you have
ANSWERED THE QUESTION.
= 49.48…m3 This question is not finished
Vol roof = 1/3π r2h= π × (1.5) 2 × 2.5 ÷ 3 until you have calculated
TOTAL VOLUME.
= 5.89…m3
3. Add to find total volume.
Total Volume = 49.48.. + 5.89..
Next Comment
= 55.37…m 3

Volumes Menu
So Volume is 55 m3
Back to Home
VOLUMES: Question 3B
A metal pencil sharpener is made by drilling a conical hole in a
small block of metal. The block has a square end of side 1.6cm and is
3.4cm long. The cone is also 3.4cm long with a diameter of 1.2cm.
Find the volume of metal used to 2 significant figures.

Get hint

Reveal answers only

1.2cm Go to full solution


3.4cm
Go to Comments

1.6cm
Go to Volumes Menu

EXIT
VOLUMES: Question 3B
A metal pencil sharpener is made by drilling a conical hole in a
small block of metal. The block has a square end of side 1.6cm and is
3.4cm long. The cone is also 3.4cm long with a diameter of 1.2cm.
Find the volume of metal used to 2 significant figures.
Vcone = 13 π r 2 h

What would you like to do now?

Reveal answers only

1. Volume2.ofSubtract
1.2cm Go to full solution
cuboid: volume of
3.4cm cone:
3. Remember
V = L × BV× =H1 πtoruse 2
h Go to Comments
3
RADIUS.
1.6cm
Go to Volumes Menu

EXIT
VOLUMES: Question 3B
A metal pencil sharpener is made by drilling a conical hole in a
small block of metal. The block has a square end of side 1.6cm and is
3.4cm long. The cone is also 3.4cm long with a diameter of 1.2cm.
Find the volume of metal used to 2 significant figures.
= 7.4 cm3

What would you like to do now?

1.2cm Go to full solution


3.4cm
Go to Comments

1.6cm
Go to Volumes Menu

EXIT
Question 3B 1. Make sure you use radius.
A metal pencil sharpener
is made by drilling a conical d = 1.2 m so r = 0.6m
hole in a small block of metal.
The block has a square end of 2. Use formulae to find volume of
side 1.6cm and is 3.4cm long. each part of structure .
The cone is also 3.4cm long
with a diameter of 1.2cm. Vol block = lxbxh = 1.6 ×1.6 × 3.4
= 8.704…cm3
Find the volume of metal
used to 2 significant figures. Vol cone = 1/3π r2h= π × (0 ⋅6)2 ×3 ⋅4 ÷3
=1.281…cm3
3. Subtract to find total volume.
Total Volume = 8.704... - 1.281..

Continue Solution
= 7.422…cm3

Comments So Volume is 7.4 cm3

Volumes Menu
What would you like to do now?

Back to Home
Comments

You SHOULD KNOW how


1. Make sure you use radius. to find volume of a cuboid:
d = 1.2 m so r = 0.6m

2. Use formulae to find volume of height


each part of structure .
Vol block = lxbxh = 1.6 ×1.6 × 3.4 th
= 8.704…cm 3
a d
e
length br
Vol cone = 1/3π r2h= π × (0 ⋅6) ×3 ⋅4 ÷3
2

=1.281…cm3 V=lxbxh
3. Subtract to find total volume.
Total Volume = 8.704... - 1.281..
Next Comment
= 7.422…cm3
Volumes Menu
So Volume is 7.4 cm 3

Back to Home
Comments

1. Make sure you use radius. Refer to formula sheet :

d = 1.2 m so r = 0.6m
Vcone = π r h
1
3
2

2. Use formulae to find volume of


each part of structure .
Vol block = lxbxh = 1.6 ×1.6 × 3.4 height h
= 8.704…cm 3

Vol cone = 1/3π r2h= π × (0 ⋅6)2 ×3 ⋅4 ÷3


=1.281…cm3
3. Subtract to find total volume. radius r
Total Volume = 8.704... - 1.281..
Next Comment
= 7.422…cm3
Volumes Menu
So Volume is 7.4 cm 3

Back to Home
Comments

For cylinders, cones and


1. Make sure you use radius. spheres make sure you identify
the radius.
d = 1.2 m so r = 0.6m Often exam questions
give dimensions in terms of the
2. Use formulae to find volume of diameter.
each part of structure .
Vol block = lxbxh = 1.6 ×1.6 × 3.4 Make sure you have
ANSWERED THE QUESTION.
= 8.704…cm3
This question is not finished
Vol cone = 1/3π r2h= π × (0 ⋅6)2 ×3 ⋅4 ÷3 until you have calculated
=1.281…cm3 NET VOLUME.
3. Subtract to find total volume.
Total Volume = 8.704... - 1.281..
Next Comment
= 7.422…cm3
Volumes Menu
So Volume is 7.4 cm 3

Back to Home
VOLUMES: Question 4
The tank in a cold water dispenser is 0.5m high with a diameter of
0.3m. The dispenser uses small paper drinking cones. These cones
have a diameter of 8cm and a depth of 10cm.
How many full cones does a full tank fill?

10cm
8cm

Get hint
0.5m
Reveal answers only

Go to full solution
0.3m

Go to Comments

Go to Volumes Menu
EXIT
VOLUMES: Question 4
The tank in a cold water dispenser is 0.5m high with a diameter of
0.3m. The dispenser uses small paper drinking cones. These cones
have a diameter of 8cm and a depth of 10cm.
How many full cones does a full tank fill?

10cm
8cm
4. Divide volume
of cylinder by What would you like to do now?
0.5m
volume of each
2. Volume of
1. Volume
cone. Round
cylinder: cone:
of Reveal answers only Vcone = π r h
1
3
2

answer
3. Rememberin 2
V =V π =2 hto3 πuse
r0.3m
context.
1
r h Go to full solution
RADIUS. Vcylinder = π r 2 h
Go to Comments

Go to Volumes Menu
EXIT
VOLUMES: Question 4
The tank in a cold water dispenser is 0.5m high with a diameter of
0.3m. The dispenser uses small paper drinking cones. These cones
have a diameter of 8cm and a depth of 10cm.
How many full cones does a full tank fill?
= 210 cones
10cm
8cm

0.5m What would you like to do now?

Try another like this

Go to full solution
0.3m

Go to Comments

Go to Volumes Menu
EXIT
Question 4 1. Make sure you use radius.
Units must be
The tank in a cold water DISPENSER : same for all
dispenser is 0.5m high with a
diameter of 0.3m. The d = 0.3 m = 30 cm so r = 15 cm
dispenser uses small paper
CONE : d = 8 cm so r = 4 cm
drinking cones. These cones
have a diameter of 8cm and a 2. Use formulae to find volume of
depth of 10cm. each structure .
How many full cones does a Vol dispenser = π r2h = π × (15) 2 × 50
full tank fill?
= 35342.9… cm3
What would you like to do now? Vol cone = /3π r h
1 2
= π × (4)2 ×10 ÷3
= 167.55…cm3
Continue Solution
3. Divide bigger volume by smaller

Try another like this Cones filled = 35342.9.. ÷ 167.55….


Comments = 210.93…

Volumes Menu
So 210 cones can be filled.

Back to Home
Comments

1. Make sure you use radius. When calculating volumes


Units must be make sure all dimensions have
DISPENSER :
same for all the same units:
d = 0.3 m = 30 cm so r = 15 cm ALL are mm, cm, m km etc.

CONE : d = 8 cm so r = 4 cm (It is often easier to work with


2. Use formulae to find volume of smallest unit.)
each structure .
Vol dispenser = π r2h = π × (15) 2 × 50
= 35342.9… cm3
Vol cone = 1/3π r2h = π × (4)2 ×10 ÷3
= 167.55…cm3
3. Divide bigger volume by smaller

Cones filled = 35342.9.. ÷ 167.55…. Next Comment


= 210.93… Volumes Menu
So 210 cones can be filled.
Back to Home
Comments

1. Make sure you use radius. Refer to formula sheet :


Units must be
DISPENSER :
same for all
d = 0.3 m = 30 cm so r = 15 cm Vcylinder = π r 2 h
CONE : d = 8 cm so r = 4 cm
radius r
2. Use formulae to find volume of
each structure .
Vol dispenser = π r2h = π × (15) 2 × 50
= 35342.9… cm3 height h
Vol cone = 1/3π r2h = π × (4)2 ×10 ÷3
= 167.55…cm3
3. Divide bigger volume by smaller

Cones filled = 35342.9.. ÷ 167.55…. Next Comment


= 210.93… Volumes Menu
So 210 cones can be filled.
Back to Home
Comments

1. Make sure you use radius. Refer to formula sheet :


Units must be
DISPENSER :
same for all
d = 0.3 m = 30 cm so r = 15 cm Vcone = 13 π r 2 h
CONE : d = 8 cm so r = 4 cm
2. Use formulae to find volume of
each structure .
height h
Vol dispenser = π r2h = π × (15) × 50
2

= 35342.9… cm3
Vol cone = 1/3π r2h = π × (4)2 ×10 ÷3
= 167.55…cm3 radius r
3. Divide bigger volume by smaller

Cones filled = 35342.9.. ÷ 167.55…. Next Comment


= 210.93… Volumes Menu
So 210 cones can be filled.
Back to Home
Comments
For cylinders, cones and
1. Make sure you use radius. spheres make sure you identify
Units must be the radius.
DISPENSER :
same for all Often exam questions
d = 0.3 m = 30 cm so r = 15 cm give dimensions in terms of the
diameter.
CONE : d = 8 cm so r = 4 cm
Make sure you have
2. Use formulae to find volume of ANSWERED THE QUESTION.
each structure . This question is not finished
Vol dispenser = π r2h = π × (15) × 50 until you have calculated how
2
many full cones can be obtained
= 35342.9… cm 3
from cylinder.
Vol cone = 1/3π r2h = π × (4)2 ×10 ÷3 When asked how many of a
= 167.55…cm3 small shape we can get from a
big shape DIVIDE VOLUMES.
3. Divide bigger volume by smaller

Cones filled = 35342.9.. ÷ 167.55…. Next Comment


= 210.93… Volumes Menu
So 210 cones can be filled.
Back to Home
VOLUMES: Question 4B
The tank in a cold water dispenser is 0.6m high with a diameter of
0.4m. The dispenser uses small paper drinking cones. These cones
have a diameter of 7cm and a depth of 11cm.
How many full cones does a full tank fill?

11cm
7cm

Get hint
0.6m
Reveal answers only

Go to full solution
0.4m

Go to Comments

Go to Volumes Menu
EXIT
VOLUMES: Question 4B
The tank in a cold water dispenser is 0.6m high with a diameter of
0.4m. The dispenser uses small paper drinking cones. These cones
have a diameter of 7cm and a depth of 11cm.
How many full cones does a full tank fill?

11cm
7cm
4. Divide volume
of cylinder by What would you like to do now?
0.5m
0.6m
volume of each
2. Volume of
1. Volume of
cone. Round
cylinder: cone:
Reveal answers only Vcone = π r h
1
3
2

answer in 2
3. Remember
=2 πr h
V =V
1
π r0.3m
hto3 use
context.
0.4m
Go to full solution
RADIUS. Vcylinder = π r 2 h
Go to Comments

Go to Volumes Menu
EXIT
VOLUMES: Question 4B
The tank in a cold water dispenser is 0.6m high with a diameter of
0.4m. The dispenser uses small paper drinking cones. These cones
have a diameter of 7cm and a depth of 11cm.
How many full cones does a full tank fill?
= 534 cones
11cm
7cm

0.6m
What would you like to do now?

Go to full solution
0.4m

Go to Comments

Go to Volumes Menu
EXIT
Question 4B 1. Make sure you use radius.
Units must be
The tank in a cold water DISPENSER : same for all
dispenser is 0.6m high with a
diameter of 0.4m. The d = 0.4 m = 40 cm so r = 20 cm
dispenser uses small paper
CONE : d = 7 cm so r = 3.5 cm
drinking cones. These cones
have a diameter of 7cm and a 2. Use formulae to find volume of
depth of 11cm. each structure .
How many full cones does a Vol dispenser = π r2h = π × (20) 2 × 60
full tank fill?
= 75398.22…cm3
What would you like to do now? Vol cone = 1
/3 π r 2
h = π × (3 ⋅5)2
×11 ÷3
= 141.10…cm3
3. Divide bigger volume by smaller
Continue Solution Cones filled = 75398.22.. ÷ 141.10….
Comments = 534.32…

Volumes Menu
So 534 cones can be filled.

Back to Home
Comments

1. Make sure you use radius. When calculating volumes


Units must be make sure all dimensions have
DISPENSER : same for all the same units:
d = 0.4 m = 40 cm so r = 20 cm ALL are mm, cm, m km etc.

CONE : d = 7 cm so r = 3.5 cm (It is often easier to work with


2. Use formulae to find volume of smallest unit.)
each structure .
Vol dispenser = π r2h = π × (20) × 60
2

= 75398.22…cm3
Vol cone = 1/3π r2h = π × (3 ⋅5)2 ×11 ÷3
= 141.10…cm3
3. Divide bigger volume by smaller

Cones filled = 75398.22.. ÷ 141.10…. Next Comment


= 534.32… Volumes Menu
So 534 cones can be filled.
Back to Home
Comments

1. Make sure you use radius. Refer to formula sheet :


Units must be
DISPENSER : same for all
d = 0.4 m = 40 cm so r = 20 cm
Vcylinder = π r 2 h
CONE : d = 7 cm so r = 3.5 cm
radius r
2. Use formulae to find volume of
each structure .
Vol dispenser = π r2h = π × (20) × 60
2

= 75398.22…cm3 height h
Vol cone = 1/3π r2h = π × (3 ⋅5)2 ×11 ÷3
= 141.10…cm3
3. Divide bigger volume by smaller

Cones filled = 75398.22.. ÷ 141.10…. Next Comment


= 534.32… Volumes Menu
So 534 cones can be filled.
Back to Home
Comments

1. Make sure you use radius.


Units must be Refer to formula sheet :
DISPENSER : same for all
d = 0.4 m = 40 cm so r = 20 cm Vcone = 13 π r 2 h
CONE : d = 7 cm so r = 3.5 cm
2. Use formulae to find volume of
each structure .
height h
Vol dispenser = π r2h = π × (20) × 60
2

= 75398.22…cm3
Vol cone = 1/3π r2h = π × (3 ⋅5)2 ×11 ÷3
= 141.10…cm3 radius r
3. Divide bigger volume by smaller

Cones filled = 75398.22.. ÷ 141.10…. Next Comment


= 534.32… Volumes Menu
So 534 cones can be filled.
Back to Home
Comments
For cylinders, cones and
1. Make sure you use radius. spheres make sure you identify
Units must be the radius.
DISPENSER : same for all Often exam questions
d = 0.4 m = 40 cm so r = 20 cm give dimensions in terms of the
diameter.
CONE : d = 7 cm so r = 3.5 cm
Make sure you have
2. Use formulae to find volume of ANSWERED THE QUESTION.
each structure . This question is not finished
Vol dispenser = π r2h = π × (20) × 60
2 until you have calculated how
many full cones can be obtained
= 75398.22…cm3 from cylinder.
Vol cone = 1/3π r2h = π × (3 ⋅5)2 ×11 ÷3 When asked how many of a
small shape we can get from a
= 141.10…cm3
big shape DIVIDE VOLUMES.
3. Divide bigger volume by smaller

Cones filled = 75398.22.. ÷ 141.10…. Next Comment


= 534.32… Volumes Menu
So 534 cones can be filled.
Back to Home
VOLUMES: Question 5
A frying pan is basically the shape of partial cone
with dimensions shown in the diagram.
Find its volume correct to 1 significant figure.

Get hint

30cm Reveal answers only

4cm
20cm Go to full solution

Go to Comments

20cm Go to Volumes Menu

EXIT
VOLUMES: Question 5
A frying pan is basically the shape of partial cone
with dimensions shown in the diagram.
Find its volume correct to 1 significant figure.

What would you like to do now?


Vcone = π r h
1
3
2

30cm Reveal answers only

4cm
20cm Go to full solution
2.two
1. Identify Volume of
possible cone: Go to Comments

π
cones. V =3.1Remember2
4.rVolume
h
20cm 3 to volume
of pan =
Go to Volumes Menu
use of big
RADIUS.
cone – volume of
small cone.

EXIT
VOLUMES: Question 5
A frying pan is basically the shape of partial cone
with dimensions shown in the diagram.
Find its volume correct to 1 significant figure.

V = 3000 cm3

30cm What would you like to do now?

4cm
20cm
Go to full solution

Go to Comments
20cm
Go to Volumes Menu

EXIT
Question 5 1. Make sure you use radius.
30cm
LARGE CONE: d = 30 cm so r = 15cm
4cm
20cm SMALL CONE: d = 20 cm so r = 10cm
2. Use formulae to find volume of
each part of structure .

20cm Vol = 1/3π r2h = π x 15 x 15 x 20 ÷ 3


cm3 = 4712.38…cm3
Vol = 1/3π r2h = π x 10 x 10 x 16 ÷ 3 cm3
Find the volume correct to
= 1675.51…cm3
1 significant figure 3. Subtract to find shaded volume
Volume of pan = 4712.38.. – 1675.51..
Continue Solution
= 3036.87…cm3

Comments
So Volume is 3000 cm3

Volumes Menu
What would you like to do now?

Back to Home
Comments
For cylinders, cones and
spheres make sure you identify
1. Make sure you use radius.
the radius.
LARGE CONE: d = 30 cm so r = 15cm Often exam questions
SMALL CONE: d = 20 cm so r = 10cm give dimensions in terms of the
diameter.
2. Use formulae to find volume of
each part of structure .
Vol = 1/3π r2h = π x 15 x 15 x 20 ÷ 3
cm3 = 4712.38…cm3
Vol = 1/3π r2h = π x 10 x 10 x 16 ÷ 3 cm3
= 1675.51…cm3
3. Subtract to find shaded volume
Volume of pan = 4712.38.. – 1675.51..
= 3036.87…cm3 Next Comment

So Volume is 3000 cm3 Volumes Menu

Back to Home
Comments

Refer to formula sheet :


1. Make sure you use radius.
LARGE CONE: d = 30 cm so r = 15cm
SMALL CONE: d = 20 cm so r = 10cm
Vcone = 13 π r 2 h
2. Use formulae to find volume of
each part of structure .
Vol = 1/3π r2h = π x 15 x 15 x 20 ÷ 3 height h
cm3 = 4712.38…cm3
Vol = 1/3π r2h = π x 10 x 10 x 16 ÷ 3 cm3
= 1675.51…cm3
3. Subtract to find shaded volume radius r
Volume of pan = 4712.38.. – 1675.51..
= 3036.87…cm3 Next Comment

So Volume is 3000 cm3 Volumes Menu

Back to Home
Comments

Must find the radius and height


1. Make sure you use radius. of the two cones:
LARGE CONE: d = 30 cm so r = 15cm
Large cone
SMALL CONE: d = 20 cm so r = 10cm
2. Use formulae to find volume of
each part of structure . height 20 cm

Vol = 1/3π r2h = π x 15 x 15 x 20 ÷ 3


cm3 = 4712.38…cm3
Vol = 1/3π r2h = π x 10 x 10 x 16 ÷ 3 cm3 radius 15 cm
= 1675.51…cm3
3. Subtract to find shaded volume
Volume of pan = 4712.38.. – 1675.51..
= 3036.87…cm3 Next Comment

So Volume is 3000 cm3 Volumes Menu

Back to Home
Comments

Must find the radius and height


1. Make sure you use radius. of the two cones:
LARGE CONE: d = 30 cm so r = 15cm
Small cone
SMALL CONE: d = 20 cm so r = 10cm
2. Use formulae to find volume of
each part of structure . height 16 cm

Vol = 1/3π r2h = π x 15 x 15 x 20 ÷ 3


cm3 = 4712.38…cm3
radius 10 cm
Vol = /3π r h = π x 10 x 10 x 16 ÷ 3 cm
1 2 3

= 1675.51…cm3
3. Subtract to find shaded volume
Volume of pan = 4712.38.. – 1675.51..
= 3036.87…cm3 Next Comment

So Volume is 3000 cm3 Volumes Menu

Back to Home
Comments

Make sure you have


1. Make sure you use radius.
ANSWERED THE QUESTION.
LARGE CONE: d = 30 cm so r = 15cm This question is not finished
SMALL CONE: d = 20 cm so r = 10cm until you have calculated
NET VOLUME.
2. Use formulae to find volume of
each part of structure .
Vol = 1/3π r2h = π x 15 x 15 x 20 ÷ 3
cm3 = 4712.38…cm3
Vol = 1/3π r2h = π x 10 x 10 x 16 ÷ 3 cm3
= 1675.51…cm3
3. Subtract to find shaded volume
Volume of pan = 4712.38.. – 1675.51..
= 3036.87…cm3 Next Comment

So Volume is 3000 cm3 Volumes Menu

Back to Home
VOLUMES: Question 6
An ice cream container has an oblong lid which consists of a
rectangle 15cm long by 10cm wide with semicircular ends.
How deep should the container be to contain 2 litres of ice cream?
Answer to 1 decimal place!

Get hint

Reveal answers only

Go to full solution

Go to Comments
15cm
Go to Volumes Menu

10cm

EXIT
VOLUMES: Question 6
An ice cream container has an oblong lid which consists of a
rectangle 15cm long by 10cm wide with semicircular ends.
How deep should the container be to contain 2 litres of ice cream?
Answer to 1 decimal place!

What would you like to do now?

2. Do not Reveal answers only


1. Break base/ round
end into basicanswers until Go to full solution
parts. Find area end.
of each part then Go to Comments
add for total area 15cm
of base. 3. Volume = desired
4. Convert
Area of base to cm3 Go to Volumes Menu
volume
x depth 3
10cm(100cm = 1 litre).
Make equation
and sole for
EXIT depth.
VOLUMES: Question 6
An ice cream container has an oblong lid which consists of a
rectangle 15cm long by 10cm wide with semicircular ends.
How deep should the container be to contain 2 litres of ice cream?
Answer to 1 decimal place!
d = 8.8 cm
What would you like to do now?

Go to full solution

Go to Comments
15cm
Go to Volumes Menu

10cm

EXIT
Question 6 1. Make sure you use radius.
15cm d = 10 cm so r = 5cm

2. Calculate area of circle and


10cm rectangle independently.

Area rect = lb = 15 x 10 = 150cm2

How deep should the container


Area circ = π r2 = π x 5 x 5

be to contain 2 litres of ice cream? = 78.539..cm2


Answer to 1 decimal place! 3. Add to find total base area.
Total area = 150 + 78.539= 228.439..cm2

Continue Solution

Comments

Volumes Menu

Back to Home
Question 6 4. Convert desired volume to cm3
15cm ( 1litre = 1000 cm3).
Desired Volume = 2 litres = 2000 cm3
10cm 5. Use fact that Volume = base area x
depth to find depth which gives
desired volume.

How deep should the container Vol = area x depth


be to contain 2 litres of ice cream? So depth = vol ÷ area
Answer to 1 decimal place! = 2000 ÷ 228.439.. cm
= 8.755.. cm
Continue Solution = 8.8cm

So depth is 8.8 cm
Comments

Volumes Menu
What would you like to do now?

Back to Home
Comments
15cm

1. Make sure you use radius.


d = 10 cm so r = 5cm 10cm

2. Calculate area of circle and


rectangle independently.

Area rect = lb = 15 x 10 = 150cm2 Volume of prism


Area circ = π r2 = π x 5 x 5 = area of base x height
= 78.539..cm2
3. Add to find total base area.
Total area = 150 + 78.539= 228.439..cm2

Next Comment

Volumes Menu

Back to Home
Comments
15cm

1. Make sure you use radius.


d = 10 cm so r = 5cm 10cm

2. Calculate area of circle and


rectangle independently.

Area rect = lb = 15 x 10 = 150cm2 Split base into a rectangle and


a circle and calculate area of each:
Area circ = π r2 = π x 5 x 5
= 78.539..cm2
3. Add to find total base area.
Total area = 150 + 78.539= 228.439..cm2

Next Comment

Volumes Menu

Back to Home
Comments
15cm

1. Make sure you use radius.


d = 10 cm so r = 5cm 10cm 15cm

2. Calculate area of circle and


rectangle independently. 10cm

Area rect = lb = 15 x 10 = 150cm2 Split base into a rectangle and


a circle:
Area circ = π r2 = π x 5 x 5 15cm

= 78.539..cm2
3. Add to find total base area. 10cm

Total area = 150 + 78.539= 228.439..cm2

Next Comment

Volumes Menu

Back to Home
Comments
15cm

1. Make sure you use radius.


d = 10 cm so r = 5cm 10cm

2. Calculate area of circle and


rectangle independently.

Area rect = lb = 15 x 10 = 150cm2 Split base into a rectangle and


a circle:
Area circ = π r2 = π x 5 x 5
= 78.539..cm2
10 cm
3. Add to find total base area.
Total area = 150 + 78.539= 228.439..cm2
radius = 5 cm
Next Comment

Volumes Menu

Back to Home
Comments
15cm

1. Make sure you use radius.


d = 10 cm so r = 5cm 10cm

2. Calculate area of circle and


rectangle independently.

Area rect = lb = 15 x 10 = 150cm2 Split base into a rectangle and


a circle and calculate area of each:
Area circ = π r2 = π x 5 x 5 15 cm
= 78.539..cm2
10 cm 10 cm
3. Add to find total base area.
Total area = 150 + 78.539= 228.439..cm2

Next Comment

Volumes Menu

Back to Home
Comments

Remember : you must find TOTAL


1. Make sure you use radius. area of base.
d = 10 cm so r = 5cm

2. Calculate area of circle and


rectangle independently.

Area rect = lb = 15 x 10 = 150cm2


Area circ = π r2 = π x 5 x 5 15 cm
= 78.539..cm2
10 cm 10 cm
3. Add to find total base area.
Total area = 150 + 78.539= 228.439..cm2

Next Comment

Volumes Menu

Back to Home
Comments

You are expected to know!!


4. Convert desired volume to cm3
( 1litre = 1000 cm3). Make sure you have
ANSWERED THE QUESTION.
Desired Volume = 2 litres = 2000 cm3
This question is not finished
5. Use fact that Volume = base area until you have calculated
x depth to find depth which gives depth which when multiplied by
desired volume. the base area just found will
Vol = area x depth equal 2000 cm3.

So depth = vol ÷ area

= 2000 ÷ 228.439.. cm
= 8.755.. cm

= 8.8cm Next Comment


So depth is 8.8 cm Volumes Menu

Back to Home
INTERMEDIATE 2 – ADDITIONAL QUESTION BANK

You have chosen to study:

UNIT 1 :
Brackets

Please choose a question to attempt from the following:

1 2 3 4 5 6

Back to
EXIT Unit 1 Menu
BRACKETS: Question 1

Simplify (3x – 7)2

Get hint

Reveal answers only

Go to full solution

Go to Comments

Go to Brackets Menu

EXIT
BRACKETS: Question 1

Simplify (3x – 7)2

1. Write
expression in
2. Make sure
full
everything in first What would you like to do now?
3. Take care
4. Remember
bracket multiplies to
with + & - signs.
everythingtidy
in up.
2nd
Reveal answers only
bracket
Go to full solution

Go to Comments

Go to Brackets Menu

EXIT
BRACKETS: Question 1

Simplify (3x – 7)2 = 9x2 – 42x + 49

What would you like to do now?

Try another like this

Go to full solution

Go to Comments

Go to Brackets Menu

EXIT
Question 1 1. Write expression in full

(3x – 7)2 = (3x – 7)(3x – 7)


Simplify (3x – 7) 2

2. Ensure everything in 2nd bracket is


multiplied by everything in 1st bracket.

= 3x(3x – 7) – 7(3x – 7)

= 9x2 – 21x – 21x + 49

= 9x2 – 42x + 49

Continue Solution What would you like to do now?


Try another like this
Comments

Brackets Menu

Back to Home
Comments

Alternative Methods

1. Write expression in full Apply Rule:

(3x – 7)2 = (3x – 7)(3x – 7) (a − b) 2 = a 2 − 2ab + b 2


2. Ensure everything in 2nd bracket is
multiplied by everything in 1st
⇒ (3x − 7) 2
bracket. = (3x) 2 − 2 × (3 x) × (7) + (7) 2
= 3x(3x – 7) – 7(3x – 7)
= 9x2 – 42x + 49
= 9x2 – 21x – 21x + 49

= 9x2 – 42x + 49

Next Comment
Brackets Menu

Back to Home
Comments

Alternative Methods

1. Write expression in full


Apply “FOIL”
(3x – 7)2 = (3x – 7)(3x – 7)
(3x – 7)2 = (3x – 7)(3x – 7)
2. Ensure everything in 2nd bracket is
multiplied by everything in 1st
bracket. = 9x2 - 21x - 21x + 49

= 3x(3x – 7) – 7(3x – 7)
Firsts Outsides Insides Lasts
= 9x – 21x – 21x + 49
2

= 9x2 – 42x + 49 NB: Remember to tidy up!

Next Comment
Brackets Menu

Back to Home
Comments

Alternative Methods

1. Write expression in full Using a Grid:

(3x – 7)2 = (3x – 7)(3x – 7) 3x -7

9x2 -21x 3x
2. Ensure everything in 2nd bracket is
multiplied by everything in 1st
bracket. -21x 49 -7
= 3x(3x – 7) – 7(3x – 7)

= 9x2 – 21x – 21x + 49 (3x – 7)2 =

= 9x2 – 42x + 49 = 9x2 – 42x + 49

Next Comment
Brackets Menu

Back to Home
BRACKETS: Question 1B

Simplify (4m – 1)2

Get hint

Reveal answers only

Go to full solution

Go to Comments

Go to Brackets Menu

EXIT
BRACKETS: Question 1B

Simplify (4m – 1)2

1. Write
expression in
2. Make sure
full
everything in first What would you like to do now?
3. Take care
4. Remember
bracket multiplies to
with + & - signs.
everythingtidy
in up.
2nd
Reveal answers only
bracket
Go to full solution

Go to Comments

Go to Brackets Menu

EXIT
BRACKETS: Question 1B

Simplify (4m – 1)2 = 16m2 – 8m + 1

What would you like to do now?

Go to full solution

Go to Comments

Go to Brackets Menu

EXIT
Question 1B 1. Write expression in full

(4m – 1)2 = (4m – 1)(4m – 1)


Simplify (4m – 1)2
2. Ensure everything in 2nd bracket is
multiplied by everything in 1st bracket.

= 4m(4m – 1) – 1(4m – 1)

= 16m2 – 4m – 4m + 1

= 16m2 – 8m + 1

Continue Solution What would you like to do now?

Comments

Brackets Menu

Back to Home
Comments

Alternative Methods

1. Write expression in full Apply Rule:

(4m – 1)2 = (4m – 1)(4m – 1) (a − b) 2 = a 2 − 2ab + b 2


2. Ensure everything in 2nd bracket is ⇒ (4m − 1) 2
multiplied by everything in 1st
bracket. = (4m) 2 − 2 × (4m) × (1) + (1) 2
= 4m(4m – 1) – 1(4m – 1)
= 16m2 – 8m + 1
= 16m2 – 4m – 4m + 1

= 16m2 – 8m + 1

Next Comment
Brackets Menu

Back to Home
Comments

Alternative Methods

1. Write expression in full


Apply “FOIL”
(4m – 1)2 = (4m – 1)(4m – 1)
(4m – 1)2 = (4m – 1)(4m – 1)

2. Ensure everything in 2nd bracket is


multiplied by everything in 1st
= 16m2 - 4m - 4m +1
bracket.
= 4m(4m – 1) – 1(4m – 1)
Firsts Outsides Insides Lasts
= 16m – 4m – 4m + 1
2

= 16m2 – 8m + 1 NB: Remember to tidy up!

Next Comment
Brackets Menu

Back to Home
Comments

Alternative Methods

1. Write expression in full Using a Grid:

4m -1
(4m – 1)2 = (4m – 1)(4m – 1)

16m2 -4m 4m
2. Ensure everything in 2 bracket is
nd

multiplied by everything in 1st


bracket. -4m 1 -1
= 4m(4m – 1) – 1(4m – 1)

= 16m2 – 4m – 4m + 1 (4m – 1)2 =

= 16m2 – 8m + 1
= 16m – 8m + 1
2

Next Comment
Brackets Menu

Back to Home
BRACKETS: Question 2

Simplify (3c - 2d) (4c + d)

Get hint

Reveal answers only

Go to full solution

Go to Comments

Go to Brackets Menu

EXIT
BRACKETS: Question 2

Simplify (3c - 2d) (4c + d)

1. Make sure
everything in first What would you like to do now?
2. Take care
3. Remember
bracket multiplies to
with + & - signs.
tidy
everything in up.
2nd
Reveal answers only
bracket
Go to full solution

Go to Comments

Go to Brackets Menu

EXIT
BRACKETS: Question 2

Simplify (3c - 2d) (4c + d) = 12c2 – 5cd – 2d2

What would you like to do now?

Try another like this

Go to full solution

Go to Comments

Go to Brackets Menu

EXIT
Question 2 1. Ensure everything in 2nd bracket is
multiplied by everything in 1st bracket.
Simplify
(3c –2d)(4c + d)
(3c - 2d) (4c + d)
= 3c(4c + d) – 2d(4c + d)

= 12c2 + 3cd – 8cd - 2d2

2. Remember to tidy up.

= 12c2 – 5cd - 2d2

Continue Solution
What would you like to do now?
Try another like this
Comments

Brackets Menu

Back to Home
Comments

Alternative Methods
1. Ensure everything in 2nd bracket is
multiplied by everything in 1st Apply “FOIL”
bracket.
(3c –2d)(4c + d) (3c – 2d)(4c + d)

= 3c(4c + d) – 2d(4c + d)
= 12c2 + 3cd - 8cd - 2d2
= 12c2 + 3cd – 8cd - 2d2

2. Remember to tidy up. Firsts Outsides Insides Lasts

= 12c2 – 5cd - 2d2


NB: Remember to tidy up!

Next Comment
Brackets Menu

Back to Home
Comments

Alternative Methods
1. Ensure everything in 2nd bracket is Using a Grid:
multiplied by everything in 1st
bracket. 3c -2d
(3c –2d)(4c + d)
12c2 -8cd 4c
= 3c(4c + d) – 2d(4c + d)
+3cd -2d2 d
= 12c + 3cd – 8cd - 2d
2 2

2. Remember to tidy up.


(3c –2d)(4c + d)
= 12c2 – 5cd - 2d2 = 12c2 – 5cd - 2d2

Next Comment
Brackets Menu

Back to Home
BRACKETS: Question 2B

Simplify (2p - q) (2p + 3q)

Get hint

Reveal answers only

Go to full solution

Go to Comments

Go to Brackets Menu

EXIT
BRACKETS: Question 2B

Simplify (2p - q) (2p + 3q)

1. Make sure
everything in first What would you like to do now?
2. Take care
3. Remember
bracket multiplies to
with + & - signs.
tidy
everything in up.
2nd
Reveal answers only
bracket
Go to full solution

Go to Comments

Go to Brackets Menu

EXIT
BRACKETS: Question 2B

Simplify (2p - q) (2p + 3q) = 4p2 + 4pq – 3q2

What would you like to do now?

Go to full solution

Go to Comments

Go to Brackets Menu

EXIT
Question 2B 1. Ensure everything in 2nd bracket is
multiplied by evrything in 1st bracket.
Simplify
(2p –q)(2p + 3q)
(2p - q) (2p + 3q)
= 2p(2p + 3q) – q(2p + 3q)

= 4p2 + 6pq – 2pq – 3q2

2. Remember to tidy up.

= 4p2 + 4pq - 3q2

Continue Solution
What would you like to do now?

Comments

Brackets Menu

Back to Home
Comments

Alternative Methods

1. Ensure everything in 2nd bracket is


multiplied by evrything in 1st Apply “FOIL”
bracket.
(2p – q)(2p + 3q)
(2p –q)(2p + 3q)

= 2p(2p + 3q) – q(2p + 3q) = 4p2 + 6pq - 2pq - 3q2

= 4p2 + 6pq – 2pq – 3q2


Firsts Outsides Insides Lasts
2. Remember to tidy up.
NB: Remember to tidy up!
= 4p2 + 4pq - 3q2

Next Comment
Brackets Menu

Back to Home
Comments

Alternative Methods

1. Ensure everything in 2nd bracket is Using a Grid:


multiplied by evrything in 1st
bracket. 2p -q

(2p –q)(2p + 3q)


4p2 -2pq 2p

= 2p(2p + 3q) – q(2p + 3q)


+6pq -3q2 +3q
= 4p2 + 6pq – 2pq – 3q2
(2p – q)(2p + 3q)
2. Remember to tidy up.
= 4p2 + 4pq - 3q2
= 4p2 + 4pq - 3q2

Next Comment
Brackets Menu

Back to Home
BRACKETS: Question 3

Simplify (3c – 2)(4c + 5) + 7c

Get hint

Reveal answers only

Go to full solution

Go to Comments

Go to Brackets Menu

EXIT
BRACKETS: Question 3

Simplify (3c – 2)(4c + 5) + 7c

2. Make sure
everything 1. in first
Think What would you like to do now?
3. Take care
bracket4. BODMAS
Remember
multiplies– to
with + & - signs.
everything tidy
deal up.
inwith
2nd
Reveal answers only
bracket.
brackets and
leave 7c until Go to full solution
end.
Go to Comments

Go to Brackets Menu

EXIT
BRACKETS: Question 3

Simplify (3c – 2)(4c + 5) + 7c = 12c2 + 14c – 10

What would you like to do now?

Try another like this

Go to full solution

Go to Comments

Go to Brackets Menu

EXIT
Question 3 1. Ensure everything in 2nd bracket is
multiplied by everything in 1st bracket.
Simplify
(3c –2)(4c + 5) + 7c
(3c – 2)(4c + 5) + 7c
= 3c(4c + 5) – 2(4c + 5) + 7c

= 12c2 + 15c – 8c - 10 + 7c

2. Remember to tidy up.

= 12c2 + 14c - 10

Continue Solution
What would you like to do now?
Try another like this
Comments

Brackets Menu

Back to Home
Comments

Alternative Methods
1. Ensure everything in 2nd bracket is
multiplied by everything in 1st LEAVE
Apply “FOIL”
bracket. for now
(3c –2)(4c + 5) + 7c
(3c – 2)(4c + 5) + 7c

= 3c(4c + 5) – 2(4c + 5) + 7c
= 12c2 + 15c - 8c - 10 + 7c
= 12c2 + 15c – 8c - 10 + 7c
Firsts Outsides Insides Lasts
2. Remember to tidy up.

= 12c2 + 14c - 10 NB: Remember to tidy up!

Next Comment
Brackets Menu

Back to Home
Comments

Alternative Methods
1. Ensure everything in 2nd bracket is
multiplied by everything in 1st Using a Grid:
bracket. 3c -2
(3c –2)(4c + 5) + 7c
12c2 -8c 4c
= 3c(4c + 5) – 2(4c + 5) + 7c
+15c -10 +5
= 12c2 + 15c – 8c - 10 + 7c

2. Remember to tidy up. (3c – 2)(4c + 5) + 7c


= 12c2 + 7c - 10 + 7c
= 12c2 + 14c - 10
= 12c2 + 14c - 10

Next Comment
Brackets Menu

Back to Home
BRACKETS: Question 3B

Simplify (5m – 2)(m + 5) – 17m

Get hint

Reveal answers only

Go to full solution

Go to Comments

Go to Brackets Menu

EXIT
BRACKETS: Question 3B

Simplify (5m – 2)(m + 5) – 17m

2. Make sure
everything 1. in first
Think What would you like to do now?
3. Take care
bracket4. BODMAS
Remember
multiplies– to
with + & - signs.
everything tidy
deal up.
inwith
2nd
Reveal answers only
bracket.
brackets and
leave – 17m Go to full solution
until end.
Go to Comments

Go to Brackets Menu

EXIT
BRACKETS: Question 3B

Simplify (5m – 2)(m + 5) – 17m = 5m2 + 6m – 10

What would you like to do now?

Go to full solution

Go to Comments

Go to Brackets Menu

EXIT
Question 3B 1. Ensure everything in 2nd bracket is
multiplied by everything in 1st bracket.
Simplify
(5m –2)(m + 5) – 17m
(5m – 2)(m + 5) – 17m
= 5m(m + 5) – 2(m + 5) – 17m

= 5m2 + 25m – 2m - 10 – 17m

2. Remember to tidy up.

= 5m2 + 6m - 10

Continue Solution
What would you like to do now?

Comments

Brackets Menu

Back to Home
Comments

Alternative Methods
1. Ensure everything in 2 bracket is
nd

multiplied by everything in 1st LEAVE


Apply “FOIL”
bracket. for now
(5m –2)(m + 5) – 17m
(5m – 2)(m + 5) - 17m

= 5m(m + 5) – 2(m + 5) – 17m


= 5m2 + 25m - 2m - 10 -17m
= 5m2 + 25m – 2m - 10 – 17m
Firsts Outsides Insides Lasts
2. Remember to tidy up.

= 5m2 + 6m - 10 NB: Remember to tidy up!

Next Comment
Brackets Menu

Back to Home
Comments

Alternative Methods
1. Ensure everything in 2 bracket is
nd

multiplied by everything in 1st Using a Grid:


bracket.
5m -2
(5m –2)(m + 5) – 17m
5m2 - 2m m
= 5m(m + 5) – 2(m + 5) – 17m
+25m -10 +5
= 5m2 + 25m – 2m - 10 – 17m

2. Remember to tidy up. (5m – 2)(m + 5) - 17m


= 5m2 + 23m - 10 -17m
= 5m2 + 6m - 10
= 5m2 + 6m - 10

Next Comment
Brackets Menu

Back to Home
BRACKETS: Question 4

Simplify (3x – 2)(4x2 + 2x - 5)

Get hint

Reveal answers only

Go to full solution

Go to Comments

Go to Brackets Menu

EXIT
BRACKETS: Question 4

Simplify (3x – 2)(4x2 + 2x - 5)

1. Make sure
everything in first What would you like to do now?
2. Take care
bracket3. Remember
multiplies to
with + & - signs.
everything tidy
in up.
2nd
Reveal answers only
bracket.
Go to full solution

Go to Comments

Go to Brackets Menu

EXIT
BRACKETS: Question 4

Simplify (3x – 2)(4x2 + 2x - 5)


= 12x3 – 2x2 - 19x + 10

What would you like to do now?

Try another like this

Go to full solution

Go to Comments

Go to Brackets Menu

EXIT
Question 4 1. Ensure everything in 2nd bracket is
multiplied by everything in 1st bracket.
Simplify
(3x – 2)(4x2 + 2x - 5)
(3x – 2)(4x2 + 2x - 5)
= 3x(4x2 + 2x – 5) – 2(4x2 + 2x – 5)

= 12x3 + 6x2 – 15x – 8x2 – 4x + 10

2. Remember to tidy up.

= 12x3 - 2x2 – 19 x + 10

Continue Solution
What would you like to do now?
Try another like this
Comments

Brackets Menu

Back to Home
Comments

Alternative Methods
1. Ensure everything in 2 bracket is
nd

multiplied by everything in 1st Using a Grid:


bracket. 4x2 + 2x -5
(3x – 2)(4x2 + 2x - 5) 3x
12x3 +6x2 -15x

-8x2 -4x +10 -2


= 3x(4x2 + 2x – 5) – 2(4x2 + 2x – 5)

= 12x3 + 6x2 – 15x – 8x2 – 4x + 10


(3x – 2)(4x2 + 2x - 5)
2. Remember to tidy up.
= 12x3 - 2x2 – 19 x + 10
= 12x - 2x – 19 x + 10
3 2

Next Comment
Brackets Menu

Back to Home
BRACKETS: Question 4B

Simplify (2v – 7)(3v2 + 2v - 3)

Get hint

Reveal answers only

Go to full solution

Go to Comments

Go to Brackets Menu

EXIT
BRACKETS: Question 4B

Simplify (2v – 7)(3v2 + 2v - 3)

1. Make sure
everything in first What would you like to do now?
2. Take care
bracket3. Remember
multiplies to
with + & - signs.
everything tidy
in up.
2nd
Reveal answers only
bracket.
Go to full solution

Go to Comments

Go to Brackets Menu

EXIT
BRACKETS: Question 4B

Simplify (2v – 7)(3v2 + 2v - 3)


= 6v3 – 17v2 – 20v + 21

What would you like to do now?

Go to full solution

Go to Comments

Go to Brackets Menu

EXIT
Question 4B 1. Ensure everything in 2nd bracket is
multiplied by everything in 1st bracket.

Simplify (2v – 7)(3v2 + 2v - 3)

(2v – 7)(3v2 + 2v - 3)
= 2v(3v2 + 2v – 3) – 7(3v2 + 2v – 3)

= 6v3 + 4v2 – 6v – 21v2 – 14v + 21

2. Remember to tidy up.

= 6v3 – 17v2 – 20v + 21

Continue Solution
What would you like to do now?

Comments

Brackets Menu

Back to Home
Comments

Alternative Methods
1. Ensure everything in 2 bracket is
nd

multiplied by everything in 1st Using a Grid:


bracket. 3v2 + 2v -3
(2v – 7)(3v2 + 2v - 3)
6v3 +4v2 - 6v 2v

= 2v(3v2 + 2v – 3) – 7(3v2 + 2v – 3) - 21v2 - 14v +21 -7

= 6v3 + 4v2 – 6v – 21v2 – 14v + 21


(2v – 7)(3v2 + 2v - 3)
2. Remember to tidy up.

= 6v3 – 17v2 – 20v + 21 = 6v3 – 17v2 – 20v + 21

Next Comment
Brackets Menu

Back to Home
BRACKETS: Question 5
The square and the rectangle have the same perimeter.
Find the value of x. (3x + 2)cm

(2x + 1)cm
(5x - 1)cm

Get hint

Reveal answers only

Go to full solution

Go to Comments

Go to Brackets Menu

EXIT
BRACKETS: Question 5
The square and the rectangle have the same perimeter.
Find the value of x. (3x + 2)cm

(2x + 1)cm
(5x - 1)cm

What would you like to do now?

Reveal answers only


1. Perimeters
2. Break all
3. Remember
same so form an to Go to full solution
brackets.
4.tidy
Solve
equation nothe
–up.
equation
units. for x. Go to Comments

Go to Brackets Menu

EXIT
BRACKETS: Question 5
The square and the rectangle have the same perimeter.
Find the value of x. (3x + 2)cm

(2x + 1)cm
(5x - 1)cm

What would you like to do now?

Try another like this


X=1
Go to full solution

Go to Comments

Go to Brackets Menu

EXIT
Question 5 (5x - 1)cm
1. Form an equation involving
expressions for perimeter of each

(5x - 1)cm
shape.
(5x - 1)cm
4(5x - 1) = 2(3x + 2) + 2(2x + 1)
2. Break all brackets.
(5x - 1)cm
Find the value of x 20x - 4 = 6x + 4 + 4x + 2
(3x + 2)cm
3. Tidy up and solve for x.

(2x + 1)cm
(2x + 1)cm 20x - 4 = 10x + 6
-10x +4 -10x +4
(3x + 2)cm
10x = 10
Continue Solution x = 1
Try another like this
What would you like to do now?
Comments

Brackets Menu

Back to Home
Comments
THINK : if I was asked to find
the perimeter of a rectangle with
1. Form an equation involving numbers what would I do?
expressions for perimeter of each
shape. ADD THE EDGES because
4(5x - 1) = 2(3x + 2) + 2(2x + 1)
Perimeter = Sum of the edges
2. Break all brackets.
The rule is the same for algebra:
20x - 4 = 6x + 4 + 4x + 2 a
3. Tidy up and solve for x.
b
20x - 4 = 10x + 6
-10x +4 -10x +4
10x = 10 Perimeter = 2a + 2b

x = 1 Next Comment
Brackets Menu

Back to Home
Comments
Work out the perimeter of each
shape using brackets where
1. Form an equation involving dimensions are duplicated.
expressions for perimeter of each
shape.
4(5x - 1) = 2(3x + 2) + 2(2x + 1)
An attempt must be made to
2. Break all brackets. form the equation.
20x - 4 = 6x + 4 + 4x + 2
Trial and error would
3. Tidy up and solve for x. receive little credit.
20x - 4 = 10x + 6
-10x +4 -10x +4
10x = 10

x = 1 Next Comment
Brackets Menu

Back to Home
BRACKETS: Question 5B
The square and the rectangle have the same perimeter.
Find the value of x. (2x + 5)cm

(x + 1)cm
(3x + 2)cm

Get hint

Reveal answers only

Go to full solution

Go to Comments

Go to Brackets Menu

EXIT
BRACKETS: Question 5B
The square and the rectangle have the same perimeter.
Find the value of x. (2x + 5)cm

(x + 1)cm
(3x + 2)cm

What would you like to do now?

Reveal answers only


1. Perimeters
2. Break all
3. Remember
same so form an to Go to full solution
brackets.
4.tidy
Solve
equation nothe
–up.
equation
units. for x. Go to Comments

Go to Brackets Menu

EXIT
BRACKETS: Question 5B
The square and the rectangle have the same perimeter.
Find the value of x. (2x + 5)cm

(x + 1)cm
(3x + 2)cm

2 What would you like to do now?


x=
3 Go to full solution

Go to Comments

Go to Brackets Menu

EXIT
Question 5B (3x + 2)cm 1. Form an equation involving
expressions for perimeter of each

(3x + 2)cm
shape.
(3x + 2)cm
4(3x + 2) = 2(2x + 5) + 2(x + 1)
2. Break all brackets.
(3x + 2)cm
Find the value of x 12x + 8 = 4x + 10 + 2x + 2
(2x + 5)cm
3. Tidy up and solve for x.

(x + 1)cm
(x + 1)cm 12x + 8 = 6x + 12
-6x -8 -6x -8
(2x + 5)cm
6x = 4
Continue Solution x= 4
6 = 2
3

Comments What would you like to do now?


Brackets Menu

Back to Home
Comments
THINK : if I was asked to find
the perimeter of a rectangle with
1. Form an equation involving numbers what would I do?
expressions for perimeter of each
shape. ADD THE EDGES because
4(3x + 2) = 2(2x + 5) + 2(x + 1) Perimeter = Sum of the edges
2. Break all brackets. The rule is the same for algebra:
12x + 8 = 4x + 10 + 2x + 2 a

3. Tidy up and solve for x.


b
12x + 8 = 6x + 12
-6x -8 -6x -8
6x = 4 Perimeter = 2a + 2b

x= 4
6 = 2
3
Next Comment
Brackets Menu

Back to Home
Comments
Work out the perimeter of each
shape using brackets where
1. Form an equation involving dimensions are duplicated.
expressions for perimeter of each
shape.
4(3x + 2) = 2(2x + 5) + 2(x + 1)
An attempt must be made to
2. Break all brackets. form the equation.

12x + 8 = 4x + 10 + 2x + 2 Trial and error would


3. Tidy up and solve for x. receive little credit.
12x + 8 = 6x + 12
-6x -8 -6x -8
6x = 4

x= 4
6 = 2
3
Next Comment
Brackets Menu

Back to Home
BRACKETS: Question 6

Show that the green area is given by 11x2 – 11x – 11 .

(5x + 2)

(2x + 3)
(3x – 1)
Get hint

Go to full solution

Go to Comments

Go to Brackets Menu

EXIT
BRACKETS: Question 6

Show that the green area is given by 11x2 – 11x – 11 .


(5x + 2)

(2x + 3)
(3x – 1)
What would you like to do now?

Try another like this


1. Find expression
2. Subtract
for 3.
the area area
Watch +&-
of Go to full solution
of 4.
each Tidy up.
smaller
signs.
shape.
shape from area Go to Comments
of bigger shape
(use brackets)
Go to Brackets Menu

EXIT
Question 6 1. Find an expression for the area of
Show that the green area is each shape.

given by 11x2 – 11x – 11 . Area Rect = (5x + 2)(3x – 1)


= 5x(3x – 1) + 2(3x – 1)
(5x + 2) = 15x2 – 5x + 6x - 2
= 15x2 + x - 2
(2x + 3)
(3x – 1) Area Sqr = (2x + 3)(2x + 3)
= 2x(2x + 3) + 3(2x + 3)
= 4x2 + 6x + 6x + 9
= 4x2 + 12x + 9

Continue Solution

Comments

Brackets Menu

Back to Home
Question 6 2. Subtract area of square from
Show that the green area is area of rectangle – using brackets.

given by 11x2 – 11x – 11 . Area Rect = 15x2 + x - 2

(5x + 2) Area Sqr = 4x2 + 12x + 9

(2x + 3) Green area


(3x – 1) = 15x2 + x – 2 - (4x2 + 12x + 9)
= 15x2 + x – 2 - 4x2 - 12x - 9
3. Get rid of brackets – watch signs –
tidy up.
Continue Solution
= 11x2 – 11x – 11 as required.
Try another like this
Comments What would you like to do now?

Brackets Menu

Back to Home
Comments

To find NET AREA


1. Find an expression for the area of
each shape. Net Area = area of larger shape
- area of smaller shape
Area Rect = (5x + 2)(3x – 1)
= 5x(3x – 1) + 2(3x – 1)
In this case :
= 15x2 – 5x + 6x - 2
= 15x2 + x - 2 Required Area
= Area of Rectangle – Area of
Area Sqr = (2x + 3)(2x + 3) Square
= 2x(2x + 3) + 3(2x + 3)
= 4x2 + 6x + 6x + 9
= 4x2 + 12x + 9
Next Comment
Brackets Menu

Back to Home
Comments

Alternative Methods

1. Find an expression for the area of


each shape. Apply “FOIL”

Area Rect = (5x + 2)(3x – 1) (5x + 2)(3x - 1)


= 5x(3x – 1) + 2(3x – 1)
= 15x2 – 5x + 6x - 2
= 15x2 - 5x +6x -2
= 15x + x - 2
2

Area Sqr = (2x + 3)(2x + 3)


Firsts Outsides Insides Lasts
= 2x(2x + 3) + 3(2x + 3)
= 4x2 + 6x + 6x + 9
NB: Remember to tidy up!
= 4x + 12x + 9
2

Next Comment
Brackets Menu

Back to Home
Comments

Alternative Methods

1. Find an expression for the area of Using a Grid:


each shape.
5x +2
Area Rect = (5x + 2)(3x – 1)
= 5x(3x – 1) + 2(3x – 1) 15x2 +6x 3x
= 15x2 – 5x + 6x - 2
= 15x2 + x - 2 -5x -2 -1

Area Sqr = (2x + 3)(2x + 3)


= 2x(2x + 3) + 3(2x + 3) (5x + 2)(3x - 1)
= 4x2 + 6x + 6x + 9 = 15x2 + x – 2
= 4x2 + 12x + 9
Next Comment
Brackets Menu

Back to Home
Comments

When subtracting area of smaller


2. Subtract area of square from shape you MUST enclose the
area of rectangle – using brackets. second area in brackets and then
Area Rect = 15x2 + x - 2 multiply out that bracket.

Remember it is the WHOLE of the


Area Sqr = 4x2 + 12x + 9
area of the second shape which is
being subtracted , not just the
Green area first part – hence the need for
brackets.
= 15x2 + x – 2 - (4x2 + 12x + 9)
= 15x2 + x – 2 - 4x2 - 12x - 9
3. Get rid of brackets – watch signs
– tidy up.
Next Comment
= 11x2 – 11x – 11 as required.
Brackets Menu

Back to Home
BRACKETS: Question 6B
Show that the green area is given by 26x – 3 .

(4x + 2)

(2x - 3)
(x + 3)
Get hint

Go to full solution

Go to Comments

Go to Brackets Menu

EXIT
BRACKETS: Question 6B
Show that the green area is given by 26x – 3 .

(4x + 2)

(2x - 3)
(x + 3)

What would you like to do now?


1. Find expression
2. Subtract
for 3.
the area area
Watch +&-
of Go to full solution
of 4.
each Tidy up.
smaller
signs.
shape.
shape from area Go to Comments
of bigger shape
(use brackets)
Go to Brackets Menu

EXIT
Question 6B 1. Find an expression for the area of
Show that the green area is each shape.

given by 26x – 3 . Area Rect = (4x + 2)(x + 3)


= 4x(x + 3) + 2(x + 3)
(4x + 2) = 4x2 + 12x + 2x + 6
= 4x2 + 14x + 6
(2x – 3)
(x + 3) Area Sqr = (2x - 3)(2x - 3)
= 2x(2x - 3) - 3(2x - 3)
= 4x2 – 6x – 6x + 9
= 4x2 – 12x + 9

Continue Solution

Comments

Brackets Menu

Back to Home
Question 6B 2. Subtract area of square from
Show that the green area is area of rectangle – using brackets.

given by 26x – 3 . Area Rect = 4x2 + 14x + 6

(4x + 2) Area Sqr = 4x2 – 12x + 9

(2x – 3) Green area


(x + 3) = 4x2 + 14x + 6 - (4x2 – 12x + 9)
= 4x2 + 14x + 6 - 4x2 + 12x – 9
3. Get rid of brackets – watch signs –
tidy up.

= 26x – 3 as required.
Continue Solution

Comments What would you like to do now?

Brackets Menu

Back to Home
Comments

To find NET AREA


1. Find an expression for the area of
each shape. Net Area = area of larger shape
- area of smaller shape
Area Rect = (4x + 2)(x + 3)
= 4x(x + 3) + 2(x + 3)
In this case :
= 4x2 + 12x + 2x + 6
= 4x2 + 14x + 6 Required Area
= Area of Rectangle – Area of
Square
Area Sqr = (2x - 3)(2x - 3)
= 2x(2x - 3) - 3(2x - 3)
= 4x2 – 6x – 6x + 9
= 4x2 – 12x + 9
Next Comment
Brackets Menu

Back to Home
Comments

Alternative Methods

1. Find an expression for the area of


each shape. Apply “FOIL”

Area Rect = (4x + 2)(x + 3) (4x + 2)(x + 3)


= 4x(x + 3) + 2(x + 3)
= 4x2 + 12x + 2x + 6
= 4x2 + 12x +2x +6
= 4x + 14x + 6
2

Area Sqr = (2x - 3)(2x - 3) Firsts Outsides Insides Lasts


= 2x(2x - 3) - 3(2x - 3)
= 4x2 – 6x – 6x + 9 NB: Remember to tidy up!
= 4x2 – 12x + 9
Next Comment
Brackets Menu

Back to Home
Comments

Alternative Methods

1. Find an expression for the area of Using a Grid:


each shape.
4x +2
Area Rect = (4x + 2)(x + 3)
= 4x(x + 3) + 2(x + 3) 4x2 +2x x
= 4x2 + 12x + 2x + 6
= 4x2 + 14x + 6 +12x +6 +3

Area Sqr = (2x - 3)(2x - 3)


(4x + 2)(x + 3)
= 2x(2x - 3) - 3(2x - 3)
= 4x2 – 6x – 6x + 9 = 4x2 + 14x + 6
= 4x2 – 12x + 9
Next Comment
Brackets Menu

Back to Home
Comments

When subtracting area of smaller


2. Subtract area of square from shape you MUST enclose the
area of rectangle – using brackets. second area in brackets and then
multiply out that bracket.
Area Rect = 4x2 + 14x + 6
Remember it is the WHOLE of the
Area Sqr = 4x – 12x + 9
2
area of the second shape which is
being subtracted , not just the
Green area first part – hence the need for
brackets.
= 4x2 + 14x + 6 - (4x2 – 12x + 9)
= 4x2 + 14x + 6 - 4x2 + 12x – 9
3. Get rid of brackets – watch signs
– tidy up.
Next Comment
= 26x – 3 as required.
Brackets Menu

Back to Home
INTERMEDIATE 2 – ADDITIONAL QUESTION BANK

You have chosen to study:

UNIT 1 :
The Circle

Please choose a question to attempt from the following:

1 2 3 4 5 6

Back to
EXIT Unit 1 Menu
The CIRCLE: Question 1
PQ is a tangent to the circle with centre O and diameter PR.
If angle SPQ = 64° then calculate the size of angle ROS.

Get hint
P

640 Reveal answers only

Q Go to full solution
O

Go to Comments

Go to Circle Menu

R S

EXIT
The CIRCLE: Question 1
PQ is a tangent to the circle with centre O and diameter PR.
If angle SPQ = 64° then calculate the size of angle ROS.

P
1. Angle
between What would you like to do now?
2. If triangle ina640
RADIUS
circle hasand
two radii
TANGENT 3. Angles
=thenin a Q
as sides
0O
triangle add to
90 .
triangle is0
180 . Reveal answers only
isosceles.
Go to full solution

Go to Comments
R S
Go to Circle Menu

EXIT
The CIRCLE: Question 1
PQ is a tangent to the circle with centre O and diameter PR.
If angle SPQ = 64° then calculate the size of angle ROS.

ROS = 520
P

640 What would you like to do now?


Q
O
Try another like this

Go to full solution

Go to Comments
R S
Go to Circle Menu

EXIT
Question 1 1. Where a radius meets a tangent
a right angle is formed.
P
Angle OPQ = 90° since the angle
640
° between a tangent and radius is right.
Q
26

O
128° Angle OPS = 90° - 64° = 26°
52° = 26° = 26°
°
26

2. Triangles with two sides which are


radii of circle are isosceles.
R S Angle OSP = 26° since triangle OSP
calculate the size of angle ROS. is formed from two radii and must be
isosceles.
Continue Solution
3. Angles in a triangle add to 1800.
Try another like this
Angle POS = 180° - 26° - 26° = 128°
Comments
As PR is straight line: Angle ROS = 520
Circle Menu
What would you like to do now?
Back to Home
Comments
Make full use of given diagram
to fill in angle properties.
1. Where a radius meets a tangent
a right angle is formed. right -angle
Angle OPQ = 90° since the angle P

between a tangent and radius is right.


Q
Angle OPS = 90° - 64° O
= 26°
2. Triangles with two sides which
are radii of circle are isosceles.
Angle OSP = 26° since triangle OSP R S
is formed from two radii and must be All radii are equal in length –
isosceles. watch for isosceles triangles
3. Angles in a triangle add to 1800. Next Comment

Angle POS = 180° - 26° - 26° The Circle Menu

As PR is straight line: Angle ROS = 520 Back to Home


The CIRCLE: Question 1B
EG is a tangent to the circle with centre O and diameter FG.

If angle EGH = 43° then calculate the size of angle OFH.

Get hint

G Reveal answers only

430
Go to full solution

O E
Go to Comments

Go to Circle Menu
H

EXIT
The CIRCLE: Question 1B
EG is a tangent to the circle with centre O and diameter FG.

If angle EGH = 43° then calculate the size of angle OFH.

1. Angle G
between What would you like to do now?
2. Angle at the
RADIUS and 430
circumference in a
TANGENT3. Angles
= is in a
semi-circle a E
triangle
90 0 O
. add to
right-angle.
1800. Reveal answers only

Go to full solution
H
Go to Comments
F
Go to Circle Menu

EXIT
The CIRCLE: Question 1B
EG is a tangent to the circle with centre O and diameter FG.

If angle EGH = 43° then calculate the size of angle OFH.

OFH = 430
G
What would you like to do now?
43 0

O E

Go to full solution

H
Go to Comments
F
Go to Circle Menu

EXIT
Question 1B 1. Where a radius meets a tangent
a right angle is formed.
G
Angle OGE = 90° since the angle
43 0

47
°
between a tangent and radius is right.
O E
Angle OGH = 90° - 43° = 47°
= 47° = 47°
2. Angle at the circumference in a
43° H
semi-circle is a right angle.
F Angle FHG = 90° since triangle FHG
calculate the size of angle OFH.
fits inside a semi-circle.
3. Angles in a triangle add to 1800.
Continue Solution
Angle OFG = 180° - 90° - 47° = 43°
Comments

Circle Menu
What would you like to do now?

Back to Home
Comments
Make full use of given diagram
1. Where a radius meets a tangent to fill in angle properties.
a right angle is formed.
right - angle
Angle OGE = 90° since the angle
between a tangent and radius is right.

Angle OGH = 90° - 43°


= 47°
2. Angle at the circumference in a
semi-circle is a right angle.
Angle FHG = 90° since triangle FHG
right - angle
fits inside a semi-circle.
3. Angles in a triangle add to 1800.
Next Comment
Angle OFG = 180° - 90° - 47° = 43° The Circle Menu

Back to Home
The CIRCLE: Question 2

The guard covering the blade on a


circular saw is formed from two
concentric sectors with common angle
110° and radii of 18cm and 14cm as
shown below. guard
14cm

18cm 110° Get hint

Reveal answers only

Go to full solution

Calculate the area of this guard to the Go to Comments


nearest unit.
Go to Circle Menu

EXIT
The CIRCLE: Question 2

The guard covering the blade on a


circular saw is formed from two
concentric sectors with common angle
110° and radii of 18cm and 14cm as
shown below.
1. Adopt a 14cm
2.strategy:
Remember area
is proportional
Shaded area = to What would you like to do now?
3.angle
Use
18cm 110°
brackets:
larger areasize.

it means
smaller you can
area.
key expressions Reveal answers only
directly into
calculator. Go to full solution

Calculate the area of this guard to the Go to Comments


nearest unit.
Go to Circle Menu

EXIT
The CIRCLE: Question 2

The guard covering the blade on a


circular saw is formed from two
concentric sectors with common angle
110° and radii of 18cm and 14cm as
shown below. guard
14cm

What would you like to do now?


18cm 110°

Try another like this

Go to full solution

Calculate the area of this guard to the Go to Comments


nearest unit.
Go to Circle Menu
area = 123 cm 2

EXIT
Question 2 1. Adopt a strategy.
14cm

Area = large sector – small sector


18cm 110°
2. Remember area is proportional to
angle size. A = 360
a
×π r2
= 110 /360 π R2 - 110 /360 π r2
3. Use brackets : it makes inputting
Calculate the area of this into calculator easier.
guard to the nearest unit.
= (110 x π x 18 x 18 ÷ 360)

– (110 x π x 14 x 14 ÷ 360)
Continue Solution
= 122.87..cm2
Try another like this
Comments = 123cm2
Circle Menu What would you like to do now?
Back to Home
Comments

To find NET AREA


1. Adopt a strategy.

Area = large sector – small sector


Net Area = area of larger shape
2. Remember area is proportional to - area of smaller shape
angle size. A = 360
a
×π r2
In this case :
= 110 /360 π R2 - 110 /360 π r2
Required Area
3. Use brackets : it makes inputting = Area of big segment
into calculator easier. – Area of small segment.

= (110 x π x 18 x 18 ÷ 360)

– (110 x π x 14 x 14 ÷ 360)

= 122.87..cm2 Next Comment


The Circle Menu
= 123cm 2

Back to Home
Comments

For area of sector use formula:


1. Adopt a strategy.

Area = large sector – small sector


A= a
360 ×π r 2

2. Remember area is proportional to


angle size. A = 360
a
×π r2
= 110 /360 π R2 - 110 /360 π r2 a˚

3. Use brackets : it makes inputting


into calculator easier.

= (110 x π x 18 x 18 ÷ 360) where a is the angle at the centre


covering the sector you are
– (110 x π x 14 x 14 ÷ 360)
finding the area of.
= 122.87..cm2 Next Comment
The Circle Menu
= 123cm 2

Back to Home
The CIRCLE: Question 2B
guard
The guard covering the blade on a pizza cutter
is basically a sector of a circle with radius 6cm
and angle 165°

165° Get hint

6cm
Reveal answers only

Go to full solution
Find the area of one side of this guard
Go to Comments
to the nearest unit.
Go to Circle Menu

EXIT
The CIRCLE: Question 2B
guard
The guard covering the blade on a pizza cutter
is basically a sector of a circle with radius 6cm
and angle 165°

1. Adopt a
2. Remember
strategy:area
is proportional
What to
fraction of What would you like to do now?
3. 165°
angleis
whole size.
shaded
part?
A= ×π r 2
6cm
a
Reveal answers only
360
Go to full solution
Find the area of one side of this guard
Go to Comments
to the nearest unit.
Go to Circle Menu

EXIT
The CIRCLE: Question 2B
guard
The guard covering the blade on a pizza cutter
is basically a sector of a circle with radius 6cm
and angle 165°

165°

6cm
What would you like to do now?

Go to full solution
Find the area of one side of this guard
Go to Comments
to the nearest unit.
Go to Circle Menu
area = 52 cm 2

EXIT
Question 2B 1. Remember area is proportional to
angle size. A = 360 × π r
a 2

Area = 165 /360 of area of full circle


165°
6cm
= 165 /360 π R2

= 165 x π x 6 x 6 ÷ 360

= 51.83..cm2

Find the area of one side of = 52cm2


this guard to the nearest unit.

What would you like to do now?


Continue Solution

Comments

Circle Menu

Back to Home
Comments

For area of sector use formula:


1. Remember area is proportional to
angle size. A = 360
a
×π r2
Area = 165
/360 of area of full circle
A= a
360 ×π r 2

= 165 /360 π R2

= 165 x π x 6 x 6 ÷ 360 a˚

= 51.83..cm2

= 52cm2
where a is the angle at the centre
covering the sector you are
finding the area of.
Next Comment
The Circle Menu

Back to Home
The CIRCLE: Question 3
The pendulum in a clock is 25cm long and
swings to and fro through an angle of 40°.
How long an arc does it trace out with each
swing? (Answer to 1 decimal place.)

Get hint
40°
Reveal answers only
25cm
Go to full solution

Go to Comments

Go to Circle Menu

EXIT
The CIRCLE: Question 3
The pendulum in a clock is 25cm long and
swings to and fro through an angle of 40°.
How long an arc does it trace out with each
swing? (Answer to 1 decimal place.)

1. Remember arc
length is
2. Remember
proportional toto What would you like to do now?
use DIAMETER 3.
angle size. 40°
for length of arc.
Reveal answers only
length = a
360 πd
×25cm
Go to full solution

Go to Comments

Go to Circle Menu

EXIT
The CIRCLE: Question 3
The pendulum in a clock is 25cm long and
swings to and fro through an angle of 40°.
How long an arc does it trace out with each
swing? (Answer to 1 decimal place.)

What would you like to do now?


40°
Try another like this
25cm
Go to full solution

Go to Comments

Go to Circle Menu

EXIT
Length = 17.5 cm
Question 3 1. Remember length is proportional to
angle size. l = a × π d
360

40° Arc length = 40 /360 of circumference

25cm = 1/9π d
2. Remember to use DIAMETER for
arc length.
= π x 50 ÷ 9
How long an arc does it trace
= 17.45....cm
out with each swing?
(Answer to 1 decimal place.) = 17.5cm

Continue Solution What would you like to do now?


Try another like this
Comments

Circle Menu

Back to Home
Comments

1. Remember length is proportional For length of arc use formula:


to angle size. l = a × π d
360 l= a
360 ×π d
Arc length = 40 /360 of circumference
= 1/9π d
2. Remember to use DIAMETER for a˚
arc length.
= π x 50 ÷ 9

= 17.45....cm
where a is the angle at the centre
= 17.5cm covering the arc you are
finding the length of.
Next Comment
The Circle Menu

Back to Home
Comments

Make sure you identify


1. Remember length is proportional the diameter.
to angle size. l = a × π d Often exam questions like this
360
give dimensions in terms of the
Arc length = 40 /360 of circumference
radius.
= 1/9π d
2. Remember to use DIAMETER for
arc length.
= π x 50 ÷ 9

= 17.45....cm

= 17.5cm

Next Comment
The Circle Menu

Back to Home
The CIRCLE: Question 3B
George of the Jungle swings on a vine that
is 15m long.
If the vine arcs through 130° then find how
far George swings to 3 significant figures.

130° 15m
Get hint

Reveal answers only

Go to full solution

Go to Comments

Go to Circle Menu

EXIT
The CIRCLE: Question 3B
George of the Jungle swings on a vine that
is 15m long.
If the vine arcs through 130° then find how
far George swings to 3 significant figures.

1. Remember arc
length is 15m
2. Remember
130°
proportional to to
What would you like to do now?
use DIAMETER 3.
angle size.
for length of arc.
Reveal answers only
length = a
360 ×π d
Go to full solution

Go to Comments

Go to Circle Menu

EXIT
The CIRCLE: Question 3B
George of the Jungle swings on a vine that
is 15m long.
If the vine arcs through 130° then find how
far George swings to 3 significant figures.

= 34.0 cm

130° 15m

What would you like to do now?

Go to full solution

Go to Comments

Go to Circle Menu

EXIT
Question 3B 1. Remember length is proportional to
angle size. l = 360 × π d
a
130° 15m
Arc length = 130 /360 of circumference
= 13 /36 π d
2. Remember to use DIAMETER for
arc length.
= π x 30 x 13 ÷ 36

= 34.03....cm
Find how far George swings
to 3 significant figures. = 34.0cm

What would you like to do now?


Continue Solution

Comments

Circle Menu

Back to Home
Comments

For length of arc use formula:


1. Remember length is proportional
to angle size. l = 360 × π d
a
l= a
360 ×π d
Arc length = 130 /360 of circumference
= 13 /36 π d
2. Remember to use DIAMETER for a˚
arc length.
= π x 30 x 13 ÷ 36

= 34.03....cm where a is the angle at the centre


covering the arc you are
= 34.0cm
finding the length of.

Next Comment
The Circle Menu

Back to Home
Comments

1. Remember length is proportional Make sure you identify


to angle size. l = 360 × π d
a the diameter.
Often exam questions like this
Arc length = 130 /360 of circumference give dimensions in terms of the
radius.
= 13 /36 π d
2. Remember to use DIAMETER for
arc length.
= π x 30 x 13 ÷ 36

= 34.03....cm

= 34.0cm

Next Comment
The Circle Menu

Back to Home
The CIRCLE: Question 4
This picture shows an arch bridge
The curved part of the bridge is
formed from an arc of a circle.

h metres
A B

60m Get hint

C
Reveal answers only
In this diagram C is the centre of
the circle.
Go to full solution
AB is the roadway between the
arch. AB is 90m long. Go to Comments
CA & CB are radii of length 60m.
Go to Circle Menu
Find h, the height of the bridge
EXIT
above the road.
The CIRCLE: Question 4
This picture shows an arch bridge
The curved part of the bridge is
formed from an arc of a circle.

h metres
A B
1. Vertical radius
What would you like to do now?
is an axis of
2.
60mUse it to 4.
symmetry. from
Subtract
3. Use
this pythagoras
two right-angled distance
toC find
triangles. fromdistance
the
from C to road Reveal answers only
In this diagram C is
vertical the centre of
surface.
the circle. radius to find
Go to full solution
h.
AB is the roadway between the
arch. AB is 90m long. Go to Comments
CA & CB are radii of length 60m.
Go to Circle Menu
Find h, the height of the bridge
EXIT
above the road.
The CIRCLE: Question 4
This picture shows an arch bridge
The curved part of the bridge is
formed from an arc of a circle.

h metres
A B

60m What would you like to do now?

C
Try another like this
In this diagram C is the centre of
the circle.
Go to full solution
AB is the roadway between the
arch. AB is 90m long. Go to Comments
CA & CB are radii of length 60m.
Go to Circle Menu
Find h, the height of the bridge
EXIT h = 20.31 m
above the road.
Question 4 1. Vertical radius is an axis of
symmetry. Use it to form 2 right-
h metres angles triangles.
A B 45m

39.69
60m xm
60m
C
x = 39.69 2. Now use pythagoras.
AB is 90m long.
Find h, the height of the bridge x2 = 602 - 452

above the road. x2 = 1575


x = 39.69
x = √1575
Continue Solution
x = 39.69

Comments

Circle Menu

Back to Home
Question 4 1. Vertical radius is an axis of
symmetry. Use it to form 2 right-
h metres angles triangles.
A B 45m

39.69
60m xm
60m
C

AB is 90m long. x = 39.69


Find h, the height of the bridge 3. Now subtract x from vertical radius
above the road. to find h.

h = 60 – 39.69 = 20.31
Continue Solution
Try another like this
Height above road is 20.31metres.

Comments What would you like to do now?


Circle Menu

Back to Home
Comments

In questions like this you must


1. Vertical radius is an axis of identify the horizontal line as
symmetry. Use it to form 2 right- a chord of the circle.
angles triangles.
45m Then you use the fact that a
chord is cut in half by the radius
which meets it at 900 (there will
xm always be one).
60m
This fact allows you to form two
2. Now use pythagoras. identical right-angled triangles.
Hence you can use “old faithful”
x2 = 602 - 452 – Pythagoras.
x2 = 1575

x = √1575
Next Comment
x = 39.69
The Circle Menu

Back to Home
Comments
Use diagram or a sketch to help
you.
1. Vertical radius is an axis of This is the diagram you should
symmetry. Use it to form 2 right- aim for:
angles triangles.
45m h

r-h
xm r
60m

2. Now use pythagoras.


x2 = 602 - 452
x2 = 1575

x = √1575
Next Comment
x = 39.69
The Circle Menu

Back to Home
Comments
This is a crucial step.
You have calculated the
3. Now subtract x from vertical distance from the Centre to the
radius to find h. chord.
The height is what’s left over
h = 60 – 39.69 = 20.31 when you subtract this distance
from the RADIUS.
Height above road is 20.31metres. h

r-h
r

Next Comment
The Circle Menu

Back to Home
The CIRCLE: Question 4B
The entrance to a tunnel on an underground
railway track is circular but has a flat base.
Details about this entrance are shown below
D

h C
Get hint

A B Reveal answers only


C is the centre of the circle
Go to full solution
AB, the base of the tunnel, is 2m
wide.
Go to Comments
CA, CB & CD are all radii of length
1.5m. Go to Circle Menu
Find h, the height of the tunnel to
EXIT
2 decimal places.
The CIRCLE: Question 4B
The entrance to a tunnel on an underground
railway track is circular but has a flat base.
Details about this entrance are shown below
D

1. Vertical radius
h C
is an axis of
2.symmetry.
Use it to 4.
fromAdd this What would you like to do now?
3. Use
two right-angled pythagoras
distance to
to the
triangles.findvertical
distance
A B Reveal answers only
from base to
radius to find C.
C is the centre of the circle
h.
Go to full solution
AB, the base of the tunnel, is 2m
wide.
Go to Comments
CA, CB & CD are all radii of length
1.5m. Go to Circle Menu
Find h, the height of the tunnel to
EXIT
2 decimal places.
The CIRCLE: Question 4B
The entrance to a tunnel on an underground
railway track is circular but has a flat base.
Details about this entrance are shown below
D

h Height of tunnel is 2.62metres.


C

A B What would you like to do now?


C is the centre of the circle
Go to full solution
AB, the base of the tunnel, is 2m
wide.
Go to Comments
CA, CB & CD are all radii of length
1.5m. Go to Circle Menu
Find h, the height of the tunnel to
EXIT
2 decimal places.
Question 4B 1. Vertical radius is an axis of
D
symmetry. Use it to form 2 right-
angles triangles.
h C 1.5m
xm

A 1.12 B
1m
AB is 2m long. x =1.12 2. Now use pythagoras.
CA, CB & CD are all radii of
length 1.5m. x2 = 1.52 - 12
Find h, the height of the tunnel,
x2 = 1.25
to 2 decimal places.
x =1.12 x = √1.25

Continue Solution x =1.1180…

Comments x =1.12

Circle Menu

Back to Home
Question 4B 1. Vertical radius is an axis of
D
symmetry. Use it to form 2 right-
angles triangles.
h C 1.5m
xm

A 1.12 B
1m
AB is 2m long.
x =1.12
CA, CB & CD are all radii of
length 1.5m. 3. Now add x to vertical radius to find
Find h, the height of the tunnel, TOTAL height of tunnel.
to 2 decimal places. h = 1.5 + 1.12 = 2.62

Continue Solution
Height of tunnel is 2.62metres.

Comments What would you like to do now?


Circle Menu

Back to Home
Comments

In questions like this you must


1. Vertical radius is an axis of identify the horizontal line as
symmetry. Use it to form 2 right- a chord of the circle.
angles triangles.
Then you use the fact that a
1.5m chord is cut in half by the radius
xm which meets it at 900 (there will
always be one).
1m
This fact allows you to form two
2. Now use pythagoras. identical right-angled triangles.
Hence you can use “old faithful”
x2 = 1.52 - 12 – Pythagoras.
x2 = 1.25

x = √1.25
Next Comment
x =1.1180…
The Circle Menu
x =1.12
Back to Home
Comments
Use diagram or a sketch to help
you.
1. Vertical radius is an axis of
This is the diagram you should
symmetry. Use it to form 2 right-
aim for:
angles triangles.

1.5m
xm

1m r
r-h
2. Now use pythagoras.

x2 = 1.52 - 12
x2 = 1.25

x = √1.25
Next Comment
x =1.1180…
The Circle Menu
x =1.12
Back to Home
Comments
This is a crucial step.
You have calculated the
3. Now add x to vertical radius to distance from the Centre to the
find TOTAL height of tunnel. chord.
The height is this distance PLUS
h = 1.5 + 1.12 = 2.62 the RADIUS.

Height of tunnel is 2.62metres.

r
r-h

Next Comment
The Circle Menu

Back to Home
The CIRCLE: Question 5
The spinning wheel in a quiz show has a diameter of 1.2m and is
divided into nine equal sectors as shown below.

A If the tip of the arrow travels


E F clockwise from E to H how far
is this?

B
I Get hint

Reveal answers only


G
D
Go to full solution

H C
Go to Comments

Go to Circle Menu

EXIT
The CIRCLE: Question 5
The spinning wheel in a quiz show has a diameter of 1.2m and is
divided into nine equal sectors as shown below.

A If the tip of the arrow travels


E F clockwise from E to H how far
is this?
1. Take note of
direction, order of B
I
2. Weand
are how
looking What would you like to do now?
letters
for a has
distance 3.
the circle been
around the outside
divided. Reveal answers only
length of=an360
– the length
a
× Gπ d
D arc. Go to full solution

H C
Go to Comments

Go to Circle Menu

EXIT
The CIRCLE: Question 5
The spinning wheel in a quiz show has a diameter of 1.2m and is
divided into nine equal sectors as shown below.

A If the tip of the arrow travels


E F clockwise from E to H how far
is this?

B
I

What would you like to do now?


G
D
Go to full solution

H C
Go to Comments

distance = 2.51 m Go to Circle Menu

EXIT
Question 5 1. Make sure you check direction,
A
order of letters and what fraction is
E F
1 2 being asked for.
3
I B Fraction = 6/9 = 2/3 .
4
2. Now calculate the required distance
remembering length = 360 × π d
a
5 G
D 6
H C dist = fraction × π d
The spinning wheel has a
diameter of 1.2m. Dist = 2/3 of circumference.
If the tip of the arrow travels = 2π d ÷ 3
clockwise from E to H how far = 2 x π x 1.2 ÷
is this? 3
= 2.513…
Continue Solution
= 2.51 metres
Comments

Circle Menu
What would you like to do now?

Back to Home
Comments
Look out for questions which
break circle into distinct
1. Make sure you check direction,
sectors.
order of letters and what fraction is
being asked for.
You can use fractions
Fraction = 6/9 = 2/3 .
OR
2. Now calculate the required
distance remembering length = a
360 ×π d

dist = fraction × π d
Dist = 2/3 of circumference.
= 2π d ÷ 3
= 2 x π x 1.2 ÷
3 Next Comment
= 2.513…
The Circle Menu
= 2.51 metres
Back to Home
Comments

1. Make sure you check direction, For length of arc use formula:
order of letters and what fraction is
being asked for. l= a
360 ×π d
Fraction = 6/9 = 2/3 .
2. Now calculate the required
distance a˚

dist = fraction × π d
Dist = 2/3 of circumference.
where a is the angle at the centre
= 2π d ÷ 3 covering the arc you are
finding the length of.
= 2 x π x 1.2 ÷
3 Next Comment
= 2.513…
The Circle Menu
= 2.51 metres
Back to Home
Comments

1. Make sure you check direction, Make sure you identify


order of letters and what fraction is the diameter.
being asked for. Often exam questions like this
give dimensions in terms of the
Fraction = 6/9 = 2/3 . radius.
2. Now calculate the required
distance remembering

dist = fraction × π d
Dist = 2/3 of circumference.
= 2π d ÷ 3
= 2 x π x 1.2 ÷
3 Next Comment
= 2.513…
The Circle Menu
= 2.51 metres
Back to Home
The CIRCLE: Question 6 A

An earring is formed from a partial


circle and triangle as shown.

Details of the figure are as follows:

AB & AD are tangents to the circle.


B D
C is the centre of the circle. 130°
16mm
C
CB & CD are radii of length 16mm.

Obtuse angle BCD is 130°. Get hint


Find the area of the earring to the
Reveal answers only
nearest mm2.

Go to full solution

Go to Comments

EXIT Go to Circle Menu


The CIRCLE: Question 6 A

An earring is formed from a partial


circle and triangle as shown.

Details of the figure are as follows:

AB & AD are tangents to the circle.


B D
C is the centre of the circle. 130°
16mm
1. The earring C
CB & CD are radiiofofa length 16mm.
consists
2. You must use
tangent kite and a 3.
Obtuse the REFLEX
angle isangle
BCD Find 130°.
major sector. What would you like to do now?
when calculating
5. Find area of
the areaof
thenArea
Find the area ofthe
4. Split
the area
each
add. =
of earring
tangent
the
one π
× r
a kite
to the
360triangle
2
2 intosector.
major 2 identical Reveal answers only
nearest mm . then double.
triangles. Use
SOHCAHTOA to find Go to full solution
missing side.
Go to Comments

EXIT Go to Circle Menu


The CIRCLE: Question 6 A

An earring is formed from a partial


circle and triangle as shown.

Details of the figure are as follows:

AB & AD are tangents to the circle.


B D
C is the centre of the circle. 130°
16mm
C
CB & CD are radii of length 16mm.

Obtuse angle BCD is 130°.


Find the area of the earring to the
What would you like to do now?
nearest mm2.

Go to full solution
distance = 2.51 m
Go to Comments

EXIT Go to Circle Menu


Question 6 A 1. Break the shape into its component
parts : tangent kite and major sector.

2. Find area of sector using REFLEX


angle. Area = 360a
×π r2
Area = 230 /360 area circle.
B D
130° = 230 x π x 162 ÷ 360
16mm
C = 513.82…mm2
Reflex
3. Split the kite in half keeping in mind
Find the area of the earring to that angles B & D are 90° .

the nearest mm2.


x mm
Continue Solution
Comments 65°
16mm
Circle Menu

Back to Home
Question 6 A 4. Now use SOHCAH TOA to find
length of tangent AB.

opp
x mm
hyp
B D
130° 65°
16mm adj 16mm
C

x = tan65°
Find the area of the earring to 16 1
x = 16tan65°
the nearest mm2.
= 34.31..mm
Continue Solution
Comments

Circle Menu

Back to Home
Question 6 A 5. Find area of triangle using
A = ½ x B x H. Then double to find
total area of kite.

Area ∆ = 16 x 34.31..÷ 2 mm2


= 274.49..mm2
B D
130°
16mm Area kite = 2 x 274.49… = 548.99..mm2
C

6. Add areas of component shapes to


find total area.
Find the area of the earring to
the nearest mm2. Area earring = 513.82..+ 548.99.. mm2
= 1062.81…mm2
Continue Solution = 1063 mm2
Comments

Circle Menu
What would you like to do now?

Back to Home
Comments
The shape is split into a sector
1. Break the shape into its and two right angled triangles:
component parts : tangent kite and A A
major sector.
2. Find area of sector using REFLEX
angle. Area = 360a
×π r2
Area = 230 /360 area circle.
B D B D
= 230 x π x 16 ÷ 360
2 130°
16mm 16mm
C
C
= 513.82…mm2

3. Split the kite in half keeping in 16mm


mind that angles B & D are 90° . 230˚

x mm Next Comment
The Circle Menu
65°
16mm
Back to Home
Comments

For area of sector use formula:


1. Break the shape into its
component parts : tangent kite and
major sector. A= a
360 ×π r 2

2. Find area of sector using REFLEX


angle. Area = 360a
×π r2
Area = 230 /360 area circle. 16mm
= 230 x π x 162 ÷ 360 a˚
= 513.82…mm2

3. Split the kite in half keeping in where a is the angle at the centre
mind that angles B & D are 90° . covering the sector you are
finding the area of.

x mm Next Comment
The Circle Menu
65°
16mm
Back to Home
Comments

You are expected to remember


from standard grade:
4. Now use SOHCAH TOA to find
length of tangent AB. opp
sin a =
o

hyp
opp
x mm
adj
hyp cos a = o

hyp
65°
adj 16mm
opp
tan a =
o

x = tan65° adj
16 1
x = 16tan65°
Next Comment
= 34.31..mm
The Circle Menu

Back to Home
Comments

You are expected to remember


5. Find area of triangle using from standard grade:
A = ½ x B x H. Then double to find
total area of kite.

height
Area ∆ = 16 x 34.31..÷ 2 mm 2

= 274.49..mm2

base
Area kite = 2 x 274.49… = 548.99..mm2
Area of triangle =
6. Add areas of component shapes ½ x base x height
to find total area.
OR (base x height) ÷2
Area earring = 513.82..+ 548.99.. mm2
= 1062.81…mm2 Next Comment

= 1063 mm2 The Circle Menu

Back to Home
Comments

Make sure you have


5. Find area of triangle using
ANSWERED THE QUESTION.
A = ½ x B x H. Then double to find
This question is not finished
total area of kite.
until you have calculated
TOTAL AREA.
Area ∆ = 16 x 34.31..÷ 2 mm2
= 274.49..mm2 Now you must add together the
areas of the individual shapes.

Area kite = 2 x 274.49… = 548.99..mm2

6. Add areas of component shapes


to find total area.

Area earring = 513.82..+ 548.99.. mm2


= 1062.81…mm2 Next Comment

= 1063 mm2 The Circle Menu

Back to Home
INTERMEDIATE 2 – ADDITIONAL QUESTION BANK

You have chosen to study:

UNIT 1 :
Factors

Please choose a question to attempt from the following:

1 2 3 4

Back to
EXIT Unit 1 Menu
FACTORS: Question 1

Factorise m2 - 4m - 21

Get hint

Reveal answers only

Go to full solution

Go to Comments

Go to Factors Menu

EXIT
FACTORS: Question 1

Factorise m2 - 4m - 21

2.
The two values we are
looking1.for satisfy the What would you like to do now?
conditions…
This is a simple
quadratic
Add up towhich–4 / multiply Reveal answers only
splits
to giveinto
-21two pairs
of brackets each
Go to full solution
starting with m.
Go to Comments

Go to Factors Menu

EXIT
FACTORS: Question 1

Factorise m2 - 4m - 21 = (m – 7) ( m + 3)

What would you like to do now?

Try another like this

Go to full solution

Go to Comments

Go to Factors Menu

EXIT
Question 1 1. This is a simple quadratic which
splits into two pairs of brackets each
Factorise starting with m.
m2 - 4m - 21 m 2 - 4m - 21

= (m - 7 ) (m + 3 )
2. The two values we are looking for
satisfy two conditions…
“Add up to –4 & multiply to give -21”

Factors of 21:

1 x 21 3x7
Continue Solution
Try another like this -7 + 3 = -4 & -7 x 3 = -21
Comments
What would you like to do now?
Factors Menu

Back to Home
Markers Comments
Factorising Simple quadratics:
1. This is a simple quadratic which
splits into two pairs of brackets each There are many methods used
starting with m. to factorise quadratics.
One of the most common
m 2 - 4m - 21
approaches is to use trial and
error.
= (m - 7 ) (m + 3 )
2. The two values we are looking for
satisfy two conditions…
“Add up to –4 & multiply to give -21”

Factors of 21:

1 x 21 3x7

-7 + 3 = -4 & -7 x 3 = -21 Next Comment

Factors Menu

Back to Home
Markers Comments
The “St. Andrew’s Cross” can
1. This is a simple quadratic which assist working.
splits into two pairs of brackets each
starting with m. Factorise: m2 - 4m - 21
m 2 - 4m - 21
Looking for numbers which
= (m - 7 ) (m + 3 ) multiply together to equal -21
2. The two values we are looking for and which “cross and add”
satisfy two conditions… to equal -4.

“Add up to –4 & multiply to give -21” -7


m
Factors of 21:
m 3
1 x 21 3x7 (m - 7)(m + 3)

-7 + 3 = -4 & -7 x 3 = -21 Next Comment

Factors Menu

Back to Home
FACTORS: Question 1B

Factorise w2 – 9w + 14

Get hint

Reveal answers only

Go to full solution

Go to Comments

Go to Factors Menu

EXIT
FACTORS: Question 1B

Factorise w2 – 9w + 14

2.
The two values we are
looking1.for satisfy the What would you like to do now?
conditions…
This is a simple
quadratic
Add up towhich–9 / multiply Reveal answers only
splits
to giveinto
+14two pairs
of brackets each
Go to full solution
starting with w.
Go to Comments

Go to Factors Menu

EXIT
FACTORS: Question 1B

Factorise w2 – 9w + 14 = (w – 7) ( w – 2 )

What would you like to do now?

Go to full solution

Go to Comments

Go to Factors Menu

EXIT
Question 1B 1. This is a simple quadratic which
splits into two pairs of brackets each
Factorise starting with w.

w2 – 9w + 14 w 2 – 9w + 14

= (w - 7 ) (w - 2 )
2. The two values we are looking for
satisfy two conditions…
“Add up to –9 & multiply to give +14”

Factors of 14:

1 x 14 2x7

Continue Solution -7 + -2 = -9 & -7 x -2 = +14


Comments
What would you like to do now?
Factors Menu

Back to Home
Markers Comments
The “St. Andrew’s Cross” can
1. This is a simple quadratic which assist working.
splits into two pairs of brackets each
starting with w. Factorise: w2 - 9w + 14
w 2 – 9w + 14
Looking for numbers which
= (w - 7 ) (w - 2 ) multiply together to equal 14
2. The two values we are looking for and which “cross and add”
satisfy two conditions… to equal -9.

“Add up to –9 & multiply to give +14” -7


w
Factors of 14:
w -2
1 x 14 2x7 (w - 7)(w - 2 )

-7 + -2 = -9 & -7 x -2 = +14 Next Comment

Factors Menu

Back to Home
FACTORS: Question 2

Factorise 16b2 – 49c2

Get hint

Reveal answers only

Go to full solution

Go to Comments

Go to Factors Menu

EXIT
FACTORS: Question 2

Factorise 16b2 – 49c2

1. Note that you


can take a square
root
2. of
Soeach
eachterm.
term is
What would you like to do now?
a perfect square
and 3.
this is a
“differencepattern
of Reveal answers only
The factorisation
is squares.”
Go to full solution
A2 – B2 = (A – B)(A + B)
Go to Comments

Go to Factors Menu

EXIT
FACTORS: Question 2

Factorise 16b2 – 49c2 = (4b – 7c) (4b + 7c)

What would you like to do now?

Try another like this

Go to full solution

Go to Comments

Go to Factors Menu

EXIT
Question 2 1. Each term is a perfect square and
so this is a “difference of two
Factorise squares”.
16b2 – 49c2
16b2 – 49c2
= (4b)2 – (7c)2

2. The factorisation pattern is


A2 – B2 = (A – B)(A + B)
where A = 4b and B = 7c

= (4b – 7c)(4b + 7c)

Continue Solution What would you like to do now?


Try another like this
Comments

Factors Menu

Back to Home
Comments

Difference of two squares.


1. Each term is a perfect square and Examples: a2 - b2
so this is a “difference of two
squares”. 4a2 - b2
16b2 – 49c2 Learn the Rule:
= (4b)2 – (7c)2
a2 - b2 = (a - b)(a + b)
2. The factorisation pattern is
A2 – B2 = (A – B)(A + B) and apply directly:
where A = 4b and B = 7c 16b2 - 49c2

= (4b – 7c)(4b + 7c) = (4b)2 - (7c)2

= (4b - 7c)(4b + 7c)


Next Comment

Factors Menu

Back to Home
FACTORS: Question 2B

Factorise 9v2 – 100w2

Get hint

Reveal answers only

Go to full solution

Go to Comments

Go to Factors Menu

EXIT
FACTORS: Question 2B

Factorise 9v2 – 100w2

1. Note that you


can take a square
root
2. of
Soeach
eachterm.
term is
What would you like to do now?
a perfect square
and 3.
this is a
“differencepattern
of Reveal answers only
The factorisation
is squares.”
Go to full solution
A2 – B2 = (A – B)(A + B)
Go to Comments

Go to Factors Menu

EXIT
FACTORS: Question 2B

Factorise 9v2 – 100w2 = (3v – 10w) (3v + 10w)

What would you like to do now?

Go to full solution

Go to Comments

Go to Factors Menu

EXIT
Question 2B 1. Each term is a perfect square and
so this is a “difference of two
Factorise squares”.
9v2 – 100w2
9v2 – 100w2
= (3v)2 – (10c)2

2. The factorisation pattern is


A2 – B2 = (A – B)(A + B)
where A = 3v and B = 10c

= (3v – 10c)(3v + 10c)

What would you like to do now?


Continue Solution

Comments

Factors Menu

Back to Home
Comments

Difference of two squares.


1. Each term is a perfect square and
so this is a “difference of two Examples: a2 - b2
squares”.
4a2 - b2
9v2 – 100w2
Learn the Rule:
= (3v)2 – (10c)2
a2 - b2 = (a - b)(a + b)
2. The factorisation pattern is
A2 – B2 = (A – B)(A + B) and apply directly:
where A = 3v and B = 10c 9v2 - 100w2

= (3v – 10c)(3v + 10c) = (3v)2 - (10w)2

= (3v - 10w)(3v + 10w)


Next Comment

Factors Menu

Back to Home
FACTORS: Question 3

Factorise 3n2 – 2n – 8

Get hint

Reveal answers only

Go to full solution

Go to Comments

Go to Factors Menu

EXIT
FACTORS: Question 3

Factorise 3n2 – 2n – 8

1. Aim for two


brackets, each
beginning with a of
2. Must factors
term What would you like to do now?
3n2involving
& 8 whichn.can
add to give -2n
and multiply to Reveal answers only
give -8.
Go to full solution

Go to Comments

Go to Factors Menu

EXIT
FACTORS: Question 3

Factorise 3n2 – 2n – 8 = (3n + 4) (n – 2)

What would you like to do now?

Try another like this

Go to full solution

Go to Comments

Go to Factors Menu

EXIT
Question 3 1. This is a tricky quadratic which
splits into two pairs of brackets each
starting with a n term.
Factorise
2. List possible factors of both 3n2 & -8
3n2 – 2n – 8 in a box and check the total when
“cross” multiplying. Our target is the
middle term.
Negative end term means
Possible factors of 3n2 : 3n & n
one factor must be negative.
Possible factors of - 8 : ±1, ±2, ±4, ±8
3n x (-1) + n x 8
3n +8
= -3n + 8n
Continue Solution n -1 = 5n
≠ -2n Try
Comments again

Factors Menu

Back to Home
Question 3 1. This is a tricky quadratic which
splits into two pairs of brackets each
starting with a n term.
Factorise
2. List possible factors of both 3n2 & -8
3n2 – 2n – 8 in a box and check the total when
“cross” multiplying. Our target is the
middle term.
Negative end term means
Possible factors of 3n2 : 3n & n
one factor must be negative.
Possible factors of - 8 : ±1, ±2, ±4, ±8
3n x (+2) + n x (-4)
3n -4
= 6n - 4n
Continue Solution n +2 = 2n
≠ -2n Try
Right number – again
Comments
wrong sign :
Factors Menu swap signs!
Back to Home
Question 3 1. This is a tricky quadratic which
splits into two pairs of brackets each
starting with a n term.
Factorise
2. List possible factors of both 3n2 & -8
3n2 – 2n – 8 in a box and check the total when
“cross” multiplying. Our target is the
middle term.
Negative end term means
Possible factors of 3n2 : 3n & n
one factor must be negative.
Possible factors of - 8 : ±1, ±2, ±4, ±8
3n x (-2) + n x (+4)
3n +4
= -6n + 4n
Continue Solution n -2 = -2n
Got it !!
Comments

Factors Menu

Back to Home
Question 3 3. Write expression in factorised form.

3n2 – 2n – 8
Factorise
3n2 – 2n – 8 = (3n + 4)(n – 2)

Negative end term means


one factor must be negative.

3n 3n x (-2) + n x (+4)
+4
= -6n + 4n
Continue Solution n -2 = -2n
Try another like this Got it !!
Comments
What would you like to do now?
Factors Menu

Back to Home
Comments
The “St. Andrew’s Cross” can
1. This is a tricky quadratic which assist working.
splits into two pairs of brackets each
starting with a n term. Factorise: 3n2 - 2n – 8

2. List possible factors of both 3n2 & -8


in a box and check the total when Looking for numbers which
“cross” multiplying. Our target is the multiply together to equal -8
middle term. and which “cross and add”
to equal -2.
Possible factors of 3n2 : 3n & n
-4
3n
Possible factors of - 8 : ±1, ±2, ±4, ±8
3n x (-1) n +2
3n -4
= -3n + 8n (3n – 4)(n + 2 )
n +2 = 5n
Next Comment
≠ -2n Try
again Factors Menu

Back to Home
FACTORS: Question 3B

Factorise 4a2 + 12a - 7

Get hint

Reveal answers only

Go to full solution

Go to Comments

Go to Factors Menu

EXIT
FACTORS: Question 3B

Factorise 4a2 + 12a - 7

1. Aim for two


brackets, each
beginning with a of
2. Must factors
term involving What would you like to do now?
4a2 & -7 whicha.can
add to give 12a
and multiply to Reveal answers only
give -7.
Go to full solution

Go to Comments

Go to Factors Menu

EXIT
FACTORS: Question 3B

Factorise 4a2 + 12a - 7 = (2a + 7) (2a – 1)

What would you like to do now?

Go to full solution

Go to Comments

Go to Factors Menu

EXIT
Question 3B 1. This is a tricky quadratic which
splits into two pairs of brackets each
starting with an a term.
Factorise
2. List possible factors of both 4a2 & -7
4a2 + 12a – 7 in a box and check the total when
“cross” multiplying. Our target is the
middle term.
Negative end term means
Possible factors of 4a2 : a, 2a & 4a
one factor must be negative.
Possible factors of - 7 : ±1, ±7
4a x (-1) + a x (+7)
4a +7
= -4a + 7a
Continue Solution a -1 = 3a
≠ 12a Try
Comments Not close so again
try changing
Factors Menu
to 2a x 2a
Back to Home
Question 3B 1. This is a tricky quadratic which
splits into two pairs of brackets each
starting with an a term.
Factorise
2. List possible factors of both 3n2 & -8
4a2 + 12a – 7 in a box and check the total when
“cross” multiplying. Our target is the
middle term.
Negative end term means
Possible factors of 4a2 : a, 2a & 4a
one factor must be negative.
Possible factors of - 7 : ±1, ±7
2a x (+1) + 2a x (-7)
2a -7
= 2a - 14a
Continue Solution 2a +1 = -12a
≠ +12a Try
Comments Right number – again
wrong sign :
Factors Menu swap signs!
Back to Home
Question 3B 1. This is a tricky quadratic which
splits into two pairs of brackets each
starting with an a term.
Factorise
2. List possible factors of both 3n2 & -8
4a2 + 12a – 7 in a box and check the total when
“cross” multiplying. Our target is the
middle term.
Negative end term means
Possible factors of 4a2 : a, 2a & 4a
one factor must be negative.
Possible factors of - 7 : ±1, ±7
2a x (-1) + 2a x (+7)
2a +7
= -2a + 14a
Continue Solution 2a -1 = 12a
Got it !!
Comments

Factors Menu

Back to Home
Question 3B 3. Write expression in factorised form.

4a2 + 12a – 7
Factorise
4a2 + 12a – 7 = (2a + 7)(2a – 1)

Negative end term means


one factor must be negative.

2a x (-1) + 2a x (+7)
2a +7
= -2a + 14a
Continue Solution 2a -1 = 12a
Got it !!
Comments What would you like to do now?
Factors Menu

Back to Home
Markers Comments
The “St. Andrew’s Cross” can
1. This is a tricky quadratic which assist working.
splits into two pairs of brackets each
starting with an a term. Factorise: 4a2 + 12a – 7

2. List possible factors of both 4a2 &


-7 in a box and check the total when Looking for numbers which
“cross” multiplying. Our target is multiply together to equal -7
the middle term. and which “cross and add”
to equal 12.
Possible factors of 4a2 : a, 2a & 4a
+7
Possible factors of - 7 : ±1, ±7 NB. Two 2a
possiblities
4a x (-1) + a x (+7) for 4a2 2a -1
4a +7
= -4a + 7a (2a + 7)(2a – 1 )
a -1 = 3a
Next Comment
≠ 12a Try
again Factors Menu

Back to Home
FACTORS: Question 4

Factorise c4 + 2c3 – 3c2

Get hint

Reveal answers only

Go to full solution

Go to Comments

Go to Factors Menu

EXIT
FACTORS: Question 4

Factorise c4 + 2c3 – 3c2

2.
After extracting
1. the
common
Look for a factor
common– What would you like to do now?
factorise the remaining
factor..
expression.
Reveal answers only

Go to full solution

Go to Comments

Go to Factors Menu

EXIT
FACTORS: Question 4

Factorise c4 + 2c3 – 3c2 = c2 (c – 1) ( c + 3)

What would you like to do now?

Try another like this

Go to full solution

Go to Comments

Go to Factors Menu

EXIT
Question 4 1. Look for any common factors.
Factorise c4 + 2c3 – 3c2
c4 + 2c3 – 3c2 = c2(c2 + 2c – 3)
= c2 (c - 1 ) (c + 3 )
2. Factorise the bracket. The two
All terms contain at least a c2. values we are looking for satisfy two
conditions…
“Add up to +2 & multiply to give -3”
Negative end term means
one factor must be negative. Factors of 3:
1x3
Continue Solution
Try another like this -1 + 3 = +2 & -1 x 3 = -3
Comments
What would you like to do now?
Factors Menu

Back to Home
Markers Comments
Two Step Problems:
1. Look for any common factors.
In harder examples it is necessary
to take two steps i.e.
c4 + 2c3 – 3c2
= c2(c2 + 2c – 3)
Common Factor First
= c (c + 3) (c – 1)
2

2. Factorise the bracket. The two Quadratic or


values we are looking for satisfy two Difference of Two Squares
conditions… Second

“Add up to +2 & multiply to give -3”


Factors of 3:
1x3
Next Comment
-1 + 3 = +2 & -1 x 3 = -3
Factors Menu

Back to Home
Markers Comments
Two Step Problems:
1. Look for any common factors.
In harder examples it is necessary
to take two steps i.e.
c4 + 2c3 – 3c2
= c2(c2 + 2c – 3)
c4 + 2c3 - 3c2 = c2(c2 + 2c - 3)
= c2 (c + 3) (c – 1)
2. Factorise the bracket. The two Common factor
values we are looking for satisfy two
conditions…
c4 + 2c3 - 3c2 = c2(c + 3)(c - 1)
“Add up to +2 & multiply to give -3”
Factors of 3: Quadratic
1x3
Next Comment
-1 + 3 = +2 & -1 x 3 = -3
Factors Menu

Back to Home
FACTORS: Question 4B

Factorise 12t3 – 75t

Get hint

Reveal answers only

Go to full solution

Go to Comments

Go to Factors Menu

EXIT
FACTORS: Question 4B

Factorise 12t3 – 75t

2.
After extracting
1. the
common
Look for a factor
common– What would you like to do now?
factorise the remaining
factor..
expression.
Reveal answers only

Go to full solution

Go to Comments

Go to Factors Menu

EXIT
FACTORS: Question 4B

Factorise 12t3 – 75t = 3t (2t – 5) ( 2t + 5)

What would you like to do now?

Go to full solution

Go to Comments

Go to Factors Menu

EXIT
Question 4B 1. Look for common factors in BOTH
the numbers and the letters.
Factorise
12t3 – 75t
12t3 – 75t = 3t (4t2 – 25)
= 3t((2t)2 – 52)

Both numbers are divisible by 2. Look to factorise the remaining


3 and both terms contain at bracket. Each term is a perfect square
least a t. and so this is a “difference of two
squares”.
3. The factorisation pattern is
A2 – B2 = (A – B)(A + B)
where A = 2t and B=5
Continue Solution
= 3t (2t – 5)(2t + 5)
Comments

Factors Menu
What would you like to do now?

Back to Home
Comments
Two Step Problems:
1. Look for common factors in BOTH
the numbers and the letters. In harder examples it is necessary
to take two steps i.e.
12t3 – 75t
(4t2 – 25)
Common Factor First
= 3t((2t)2 – 52)
Quadratic or
2. Look to factorise the remaining Difference of Two Squares
bracket. Each term is a perfect Second
square and so this is a “difference of
two squares”.
3. The factorisation pattern is
A2 – B2 = (A – B)(A + B)
where A = 2t and B=5 Next Comment

= 3t (2t – 5)(2t + 5) Factors Menu

Back to Home
Comments
Two Step Problems:
1. Look for common factors in BOTH
the numbers and the letters. In harder examples it is necessary
to take two steps i.e.
12t3 – 75t
(4t2 – 25) 12t4 + 75t = 3t(4t2 - 25)

= 3t((2t)2 – 52) Common factor

2. Look to factorise the remaining = 3t((2t)2 - (5)2)


bracket. Each term is a perfect
square and so this is a “difference of
two squares”. Difference of Two Squares

3. The factorisation pattern is = 3t(2t - 5)(2t + 5)


A2 – B2 = (A – B)(A + B)
where A = 2t and B=5 Next Comment

= 3t (2t – 5)(2t + 5) Factors Menu

Back to Home

You might also like